Site Loader

Содержание

Сила Лоренца

msimagelist>msimagelist>msimagelist>msimagelist>msimagelist>msimagelist>msimagelist>msimagelist>msimagelist>msimagelist>msimagelist>msimagelist>msimagelist>msimagelist>msimagelist>msimagelist>
msimagelist>msimagelist>msimagelist>msimagelist>msimagelist>msimagelist>msimagelist>msimagelist>msimagelist>msimagelist>msimagelist>msimagelist>msimagelist>msimagelist>msimagelist>msimagelist>msimagelist>msimagelist>msimagelist>msimagelist>msimagelist>msimagelist>msimagelist>msimagelist>msimagelist>
msimagelist>msimagelist>msimagelist>msimagelist>msimagelist>msimagelist>
msimagelist>msimagelist>msimagelist>msimagelist>msimagelist>msimagelist>msimagelist>
msimagelist>msimagelist>msimagelist>msimagelist>msimagelist>msimagelist>msimagelist>msimagelist>msimagelist>msimagelist>msimagelist>msimagelist>msimagelist>msimagelist>msimagelist>msimagelist>msimagelist>msimagelist>msimagelist>msimagelist>msimagelist>
msimagelist>msimagelist>msimagelist>msimagelist>msimagelist>msimagelist>msimagelist>msimagelist>
msimagelist>msimagelist>msimagelist>msimagelist>msimagelist>msimagelist>
msimagelist>msimagelist>msimagelist>msimagelist>msimagelist>msimagelist>msimagelist>msimagelist>msimagelist>msimagelist>msimagelist>msimagelist>msimagelist>msimagelist>msimagelist>msimagelist>msimagelist>msimagelist>msimagelist>msimagelist>msimagelist>msimagelist>msimagelist>msimagelist>msimagelist>msimagelist>msimagelist>msimagelist>msimagelist>msimagelist>msimagelist>msimagelist>msimagelist>msimagelist>msimagelist>msimagelist>msimagelist>msimagelist>msimagelist>msimagelist>msimagelist>msimagelist>msimagelist>msimagelist>msimagelist>msimagelist>msimagelist>msimagelist>msimagelist>msimagelist>msimagelist>msimagelist>msimagelist>msimagelist>msimagelist>msimagelist>msimagelist>msimagelist>msimagelist>msimagelist>msimagelist>msimagelist>msimagelist>msimagelist>msimagelist>msimagelist>msimagelist>msimagelist>msimagelist>msimagelist>msimagelist>msimagelist>msimagelist>msimagelist>msimagelist>msimagelist>msimagelist>msimagelist>msimagelist>msimagelist>msimagelist>msimagelist>msimagelist>msimagelist>msimagelist>msimagelist>msimagelist>msimagelist>msimagelist>msimagelist>msimagelist>msimagelist>msimagelist>msimagelist>msimagelist>msimagelist>msimagelist>msimagelist>msimagelist>msimagelist>msimagelist>msimagelist>msimagelist>msimagelist>msimagelist>msimagelist>msimagelist>msimagelist>msimagelist>msimagelist>msimagelist>msimagelist>msimagelist>
Адроны
Альфа-распад
Альфа-частица
Аннигиляция
Антивещество
Антинейтрон
Антипротон
Античастицы
Атом
Атомная единица массы
Атомная электростанция
Барионное число
Барионы
Бета-распад
Бетатрон
Бета-частицы
Бозе – Эйнштейна статистика
Бозоны
Большой адронный коллайдер
Большой Взрыв
Боттом. Боттомоний
Брейта-Вигнера формула
Быстрота
Векторная доминантность
Великое объединение
Взаимодействие частиц
Вильсона камера
Виртуальные частицы
Водорода атом
Возбуждённые состояния ядер
Волновая функция
Волновое уравнение
Волны де Бройля
Встречные пучки
Гамильтониан
Гамма-излучение
Гамма-квант
Гамма-спектрометр
Гамма-спектроскопия
Гаусса распределение
Гейгера счётчик
Гигантский дипольный резонанс
Гиперядра
Глюоны
Годоскоп
Гравитационное взаимодействие
Дейтрон
Деление атомных ядер
Детекторы частиц
Дирака уравнение
Дифракция частиц
Доза излучения
Дозиметр
Доплера эффект
Единая теория поля
Зарядовое сопряжение
Зеркальные ядра
Избыток массы (дефект массы)
Изобары
Изомерия ядерная
Изоспин
Изоспиновый мультиплет
Изотопов разделение
Изотопы
Ионизирующее излучение
Искровая камера
Квантовая механика
Квантовая теория поля
Квантовые операторы
Квантовые числа
Квантовый переход
Квант света
Кварк-глюонная плазма
Кварки
Коллайдер
Комбинированная инверсия
Комптона эффект
Комптоновская длина волны
Конверсия внутренняя
Константы связи
Конфайнмент
Корпускулярно волновой дуализм
Космические лучи
Критическая масса
Лептоны
Линейные ускорители
Лоренца преобразования
Лоренца сила
Магические ядра
Магнитный дипольный момент ядра
Магнитный спектрометр
Максвелла уравнения
Масса частицы
Масс-спектрометр
Массовое число
Масштабная инвариантность
Мезоны
Мессбауэра эффект
Меченые атомы
Микротрон
Нейтрино
Нейтрон
Нейтронная звезда
Нейтронная физика
Неопределённостей соотношения
Нормы радиационной безопасности
Нуклеосинтез
Нуклид
Нуклон
Обращение времени
Орбитальный момент
Осциллятор
Отбора правила
Пар образование
Период полураспада
Планка постоянная
Планка формула
Позитрон
Поляризация
Поляризация вакуума
Потенциальная яма
Потенциальный барьер
Принцип Паули
Принцип суперпозиции
Промежуточные W-, Z-бозоны
Пропагатор
Пропорциональный счётчик
Пространственная инверсия
Пространственная четность
Протон
Пуассона распределение
Пузырьковая камера
Радиационный фон
Радиоактивность
Радиоактивные семейства
Радиометрия
Расходимости
Резерфорда опыт
Резонансы (резонансные частицы)
Реликтовое микроволновое излучение
Светимость ускорителя
Сечение эффективное
Сильное взаимодействие
Синтеза реакции
Синхротрон
Синхрофазотрон
Синхроциклотрон
Система единиц измерений
Слабое взаимодействие
Солнечные нейтрино
Сохранения законы
Спаривания эффект
Спин
Спин-орбитальное взаимодействие
Спиральность
Стандартная модель
Статистика
Странные частицы
Струи адронные
Субатомные частицы
Суперсимметрия
Сферическая система координат
Тёмная материя
Термоядерные реакции
Термоядерный реактор
Тормозное излучение
Трансурановые элементы
Трек
Туннельный эффект
Ускорители заряженных частиц
Фазотрон
Фейнмана диаграммы
Фермионы
Формфактор
Фотон
Фотоэффект
Фундаментальная длина
Хиггса бозон
Цвет
Цепные ядерные реакции
Цикл CNO
Циклические ускорители
Циклотрон
Чарм. Чармоний
Черенковский счётчик
Черенковсое излучение
Черные дыры
Шредингера уравнение
Электрический квадрупольный момент ядра
Электромагнитное взаимодействие
Электрон
Электрослабое взаимодействие
Элементарные частицы
Ядерная физика
Ядерная энергия
Ядерные модели
Ядерные реакции
Ядерный взрыв
Ядерный реактор
Ядра энергия связи
Ядро атомное
Ядерный магнитный резонанс (ЯМР)

Лоренца сила — это… Что такое Лоренца сила?

  • ЛОРЕНЦА СИЛА — сила, действующая на заряж. ч цу, движущуюся в эл. магн. поле. Ф ла для Л. с. F впервые получена X. А. Лоренцем, обобщившим эксперим. данные, имеет вид: F = eE+e/c(vB), (*) где е заряд ч цы, Е напряжённость электрич. поля, В магнитная индукция, v …   Физическая энциклопедия

  • ЛОРЕНЦА СИЛА — ЛОРЕНЦА СИЛА, сила (f), действующая на заряженную частицу, движущуюся в электромагнитном поле; выражается установленной Х.А. Лоренцем в конце 19 в. формулой f= qE+q(vxB) (в СИ), где q, v заряд и скорость частицы соответственно; E напряженность… …   Современная энциклопедия

  • ЛОРЕНЦА СИЛА — сила (f), действующая на заряженную частицу, движущуюся в электромагнитном поле; выражается установленной Х. А. Лоренцем в кон. 19 в. формулой: (в СГС системе единиц), где ?, ? заряд и скорость частицы, Е напряженность электрического поля, В… …   Большой Энциклопедический словарь

  • Лоренца сила — ЛОРЕНЦА СИЛА, сила (f), действующая на заряженную частицу, движущуюся в электромагнитном поле; выражается установленной Х.А. Лоренцем в конце 19 в. формулой f= qE+q(vxB) (в СИ), где q, v заряд и скорость частицы соответственно; E напряженность… …   Иллюстрированный энциклопедический словарь

  • Лоренца сила — сила (f), действующая на заряженную частицу, движущуюся в электромагнитном поле; выражается установленной Х. А. Лоренцем в конце XIX в. формулой: (в СГС системе единиц), где e, v  заряд и скорость частицы, E  напряжённость электрического поля, B  …   Энциклопедический словарь

  • Лоренца сила —         сила, действующая на заряженную частицу, движущуюся в электромагнитном поле. Формула для Л. с. F была впервые получена Х. А. Лоренцом как результат обобщения опыта и имеет вид:          F = eE + υB].          Здесь е заряд частицы, Е… …   Большая советская энциклопедия

  • ЛОРЕНЦА СИЛА — сила, действующая со стороны заданного электромагнитного поля на движущуюся заряженную частицу. Выражение для Л. с. Fбыло впервые дано Г. Лоренцем (см. [1]): где Е напряженность электрич. поля, В магнитная индукция, V скорость заряженной частицы… …   Математическая энциклопедия

  • ЛОРЕНЦА СИЛА — сила, действующая со стороны электромагнитного поля на движущуюся заряж. частицу. Л. с. F равна F = QE + О [v, В], где О заряд частицы, Е напряжённость электрич. поля, В магнитная индукция, a v скорость частицы относительно той инерциальной… …   Большой энциклопедический политехнический словарь

  • ЛОРЕНЦА СИЛА — сила (f), действующая на заряженную частицу, движущуюся в эл. магн. поле; выражается установленной X. А. Лоренцем и кон. 19 в. ф лой: f= е(Е+1/C[vB]) (в СГС системе единиц), где е, v заряд и скорость частицы, Е напряжённость электрич. поля, В… …   Естествознание. Энциклопедический словарь

  • СИЛА — векторная величина мера механического воздействия на тело со стороны др. тел, а также интенсивности др. физ. процессов и полей. Силы бывают различными: (1) С. Ампёра сила, с которой (см.) действует на проводник с током; направление вектора силы… …   Большая политехническая энциклопедия

  • Сила лоренца — справочник студента

    Подробности Категория: Электричество и магнетизм Опубликовано 11.06.2015 18:53 Просмотров: 9352

    Сила, действующая на точечную заряженную частицу со стороны электромагнитного поля, называется силой Лоренца.

    Узнай стоимость своей работы

    Бесплатная оценка заказа!

    Теория Лоренца

    Хендрик Антон Лоренц

    В 1892 г. голландский физик-теоретик Хендрик Антон Ло́ренц опубликовал работу «Электромагнитная теория Максвелла и её применение к движущимся телам», в которой объединил теорию поля и созданную им теорию электронного строения вещества.

    Лоренц предположил, что все молекулы вещества состоят из частиц, имеющих электрический заряд. Величина этих зарядов одинакова. Но одни из них заряжены отрицательно, другие положительно.

    Все эти элементарные заряды создают микроскопические электромагнитные поля, которые описываются уравнениями Максвелла.

    Конечно, теория Лоренца имела недостатки и отличалась от современной электронной теории. Но в этой работе учёный вывел формулу силы, действующей на электрический заряд со стороны электромагнитного поля. Эту силу впоследствии назвали силой Лоренца.

    Но что же такое электрический ток? Это направленное движение электрических зарядов. И если на каждую заряженную частицу действует сила Лоренца, то на отрезок проводника с током в электромагнитном поле должна действовать сила, величина которой равна сумме всех сил Лоренца, действующих на заряды, образующие электрический ток в проводнике.

    Узнай стоимость своей работы

    Бесплатная оценка заказа!

    И такая сила была открыта задолго до Лоренца. Ещё не зная о существовании силы, действующей на отдельный электрический заряд, французский физик Мари Андре Ампер в 1820 г. описал силу, действующую со стороны электромагнитного поля на проводник с током. Её назвали силой Ампера.

    Сила Ампера

    Существование силы Ампера подтверждает простой опыт. 

    Если поместить между полюсами магнита проводник и пропустить по нему электрический ток, то можно увидеть, что проводник отклоняется от своего исходного положения. Это означает, что со стороны магнитного поля на него действует сила. Эта сила называется силой Ампера.

    Её величина определяется законом Ампера: «Со стороны магнитного поля на проводник с током действует сила, величина которой прямо пропорциональна силе тока, длине проводника в магнитном поле, модулю вектора магнитной индукции и синусу угла между вектором магнитной индукции и направление тока в проводнике».

    Математическое выражение этого закона выглядит так:

    • FA = I·l·В·sinα,
    • где I– величина тока в проводнике;
    • l– длина проводника с током в магнитном поле;
    • В – магнитная индукция;
    • α — угол между вектором магнитной индукции и направление тока в проводнике.

    Связь между силой Ампера и силой Лоренца

    Действуя на проводник с током, магнитное поле воздействует на каждую заряженную частицу, создающую этот ток. А сила Ампера действует на весь проводник. Таким образом, сила Ампера равна сумме всех сил Лоренца, действующих на проводник с током.

    1. FA= F·N
    2. где F– сила Лоренца;
    3.  N— число частиц.
    4. Отсюда F= FA /N
    5. I = nqvS
    6. N = nSl
    7. Подставив эти выражения в формулу, получим выражение для силы Лоренца в магнитном поле:
    8. F = qvBˑsinα.

    Это выражение позволяет вычислить силу Лоренца в магнитном поле. Но магнитное поле не существует отдельно. Изменяясь, вместе с электрическим полем они порождают друг друга, образуя электромагнитное поле.

    А оно в каждой точке своего пространства характеризуется напряжённостью электрического поля Еи индукцией магнитного поля В. И если электрически заряженная частица движется в электромагнитном поле, то на неё одновременно действуют и электрическое, и магнитное поле.

    Значит, величина силы Лоренца, действующая со стороны электромагнитного поля на частицу с зарядом q, движущуюся со скоростью v, зависит от этих величин:

    • F = q(E + vxB)
    • F, E, vиB) – векторные величины. 
    • vxB– векторное произведение скорости движения частицы и индукции магнитного поля.
    • Направление силы Лоренца, как и силы Ампера, определяют с помощью правила левой руки: «Если расположить ладонь левой руки таким образом, чтобы линии магнитного поля входили в неё перпендикулярно, а 4 пальца направить в сторону движения частицы с положительным зарядом, или против движения частицы с отрицательным зарядом, то отогнутый на 900 большой палец покажет направление силы Лоренца».
    • Если заряженная частица движется параллельно силовым линиям магнитного поля, то величина силы Лоренца равна нулю, так как в этом случае α = 0, следовательно, sinα = 0
    • F = qvBˑsinα= 0.
    • Если же направление движения частицы перпендикулярно силовым линиям, то частица будет двигаться по окружности радиусом r, а сила Лоренца направлена к её центру, то есть является центростремительной силой.
    • Согласно второму закону Ньютона сила Лоренца равна mv2/r.
    • Отсюда
    •  
    • При движении частицы под углом к силовым линиям её траектория представляет собой винтовую (спиральную) линию, имеющую радиус r и шаг винта h.
    • Сила Лоренца не совершает работы, так как её направление всегда перпендикулярно направлению движения заряда.

    Сила Лоренца в технике

    Основное применение сила Лоренца нашла в электротехнике.

    На явлениях электромагнитной индукции и силы Лоренца основана работа электродвигателей и генераторов. Возникая в электромагнитном поле статора, она приводит во вращение ротор.

    Воздействие силы Лоренца на электроны используют в работе электронно-лучевых трубок (кинескопов), где магнитное поле, созданное специальными катушками, изменяет траекторию электронов. С помощью этой силы можно задавать орбиту движения частиц, что позволяет применять её в ускорителях заряженных частиц.

    Источник: http://ency.info/materiya-i-dvigenie/elektrichestvo-i-magnetizm/460-sila-lorentsa

    Сила Лоренца: определение, направление, формула, применение

    Мари Ампер доказал, что при наличии электрического тока в проводнике, оказавшемся в магнитном поле, он взаимодействует с силами этого поля.

    Учитывая то, что электрический ток – это не что иное, как упорядоченное движение электронов, можно предположить, что электромагнитные поля подобным образом действуют также на отдельно взятую заряженную частицу. Это действительно так.

    На точечный заряд действует сила Лоренца, модуль которой можно вычислить по формуле.

    Определение и формула

    Хендрик Лоренц доказал, что электромагнитная индукция взаимодействует с заряженными частицами. Эти взаимодействия приводят к возникновению силы Лоренца. Рассматриваемая сила возникает под действием магнитной индукции. Она перпендикулярна вектору скорости движущейся частицы (см. рис. 1). Необходимым условием возникновения этой силы является движение электрического заряда.

    Рис. 1. Выводы Лоренца

    Обратите внимание на расположение векторов (рисунок слева, вверху). Векторы, указывающие направления скорости и силы Лоренца, лежат в одной плоскости XOY, причём они расположены под углом 90º. Вектор магнитной индукции сориентирован вдоль оси Z, перпендикулярной плоскости XOY, а значит, в выбранной системе координат он перпендикулярен к векторам силы и скорости.

    По закону Ампера:

    Учитывая, что

    (здесь j – плотность тока, q – единичный заряд, n – количество зарядов на бесконечно малую единицу длины проводника, S – сечение проводника, символом v обозначен модуль скорости движущейся частицы), запишем формулу Ампера в виде:

    Так, как nSdl – общее число зарядов в объёме проводника, то для нахождения силы, действующей на точечный заряд, разделим выражение на количество частиц:

    Модуль F вычисляется по формуле:

    Из формулы следует:

    1. Сила Лоренца приобретает максимальное значение, если угол α прямой.
    2. Если точечный заряд, например, электрон, попадает в среду однородного магнитного поля, обладая некой начальной скоростью, перпендикулярной к линиям электромагнитной индукции, тогда вектор F будет перпендикулярен к вектору скорости. На точечный заряд будет действовать центробежная сила, которая заставит его вращаться по кругу. При этом работа равняется нулю (см. рис.2).
    3. Если угол между вектором индукции и скоростью частицы не равняется 90º, тогда заряд будет двигаться по спирали. Направление вращения зависит от полярности заряда (рис. 3).

    Рис. 2. Заряженная частица между полюсами магнитов Рис. 3. Ориентация вектора в зависимости от полярности заряда

    Из рисунка 3 видно, что вектор F направлен в противоположную сторону, если знак заряда меняется на противоположный (при условии, что направления остальных векторов остаются неизменными).

    Траекторию движения частицы правильно называть винтовой линией. Радиус этой винтовой линии (циклотронный радиус) определяется перпендикулярной к полю составной начальной скорости частицы.

    Шаг винтовой линии, вдоль которой перемещается частица, определяется составной начальной скорости заряда, вошедшего в однородное магнитное поле.

    Эта составная направлена параллельно к электромагнитным линиям.

    В чём измеряется?

    Размерность силы Лоренца в международной системе СИ – ньютон (Н). Разумеется, модуль силы Лоренца настолько крохотная величина, по сравнению с ньютоном, что её записывают в виде К×10-n Н, где 0

    Источник: https://www.asutpp.ru/sila-lorentsa.html

    Сила Лоренца в магнитном поле

  • Определение силы Лоренца
  • Немного истории
  • Формула силы Лоренца
  • Правило левой руки
  • Применение силы Лоренца
  • Рекомендованная литература и полезные ссылки
  • Сила Лоренса, видео
  • Определение силы Лоренца

    Сила Лоренца представляет собой комбинацию магнитной и электрической силы на точечном заряде, который вызван электромагнитными полями.

    Или другими словами, сила Лоренца – это сила, действующая на всякую заряженную частицу, которая падает в магнитном поле с определенной скоростью.

    Ее величина зависит от величины магнитной индукции В, электрического заряда частицы q и скорости, с которой частица падает в поле – V. О том какая формула расчета силы Лоренца, а также ее практическое значение в физике читайте далее.

    Немного истории

    Первые попытки описать электромагнитную силу были сделаны еще в XVIII веке.

    Ученые Генри Кавендиш и Тобиас Майер высказали предположение, что сила на магнитных полюсах и электрически заряженных объектах подчиняется закону обратных квадратов.

    Однако экспериментальное доказательство этого факта не было полным и убедительным. Только в 1784 году Шарль Августин де Кулон при помощи своего торсионного баланса смог окончательно доказать это предположение.

    В 1820 году физиком Эрстедом был открыт факт, что на магнитную стрелку компаса действует ток вольта, а Андре-Мари Ампер в этом же году смог разработать формулу угловой зависимости между двумя токовыми элементами. По сути, эти открытия стали фундаментом современной концепции электрических и магнитных полей.

    Сама же концепция получила свое дальнейшее развитие в теориях Майкла Фарадея, особенно в его представлении о силовых линиях. Лорд Кельвин и Джеймс Максвелл дополнили теории Фарадея подробным математическим описанием.

    В частности Максвеллом было создано так званное, «уравнение поля Максвелла» – представляющее собой систему дифференциальных и интегральных уравнений, описывающих электромагнитное поле и его связь с электрическими зарядами и токами в вакууме и сплошных средах.

    Джей Джей Томпсон был первым физиком, кто попытался вывести из уравнения поля Максвелла электромагнитную силу, которые действует на движущийся заряженный объект. В 1881 году он опубликовал свою формулу F = q/2 v x B. Но из-за некоторых просчетов и неполного описания тока смещения она оказалась не совсем правильной.

    И вот, наконец, в 1895 году голландский ученый Хендрик Лоренц вывел правильную формулу, которая используется и поныне, а также носит его имя, как и та сила, что действует на летящую частицу в магнитном поле, отныне называется «силой Лоренца».

    Хендрик Лоренц.

    Формула силы Лоренца

    Формула для расчета силы Лоренца выглядит следующим образом:

    Где q – электрический заряд частицы, V – ее скорость, а B – величина магнитной индукции магнитного поля.

    При этом поле B выступает в качестве силы, перпендикулярной к направлению вектора скорости V нагрузок и направлению вектора B. Это можно проиллюстрировать на диаграмме:

    Правило левой руки

    Правило левой руки позволяет физикам определять направление и возврат вектора магнитной (электродинамической) энергии.

    Представьте себе, что наша левая рука расположена таким образом, что линии магнитного поля направлены перпендикулярно внутренней поверхности руки (так, что они проникают внутрь руки), а все пальцы за исключением большого указывают на направление протекания положительного тока, отклоненный большой палец указывает на направление электродинамической силы, действующий на положительный заряд, помещенный в это поле.

    Вот так это будет выглядеть схематически.

    Есть также и второй способ определения направления электромагнитной силы. Он заключается в расположении большого, указательного и среднего пальцев под прямым углом. В этом случае указательный палец будет показывать направление линий магнитного поля, средний – направление движение тока и большой – направление электродинамической силы.

    Применение силы Лоренца

    Сила Лоренца и ее расчеты имеет свое практическое применение при создании как специальных научных приборов – масс-спектрометров, служащих для идентификации атомов и молекул, так и создании многих других устройств самого разнообразного применения. Среди устройств есть и электродвигатели, и громкоговорители, и рельсовые пистолеты.

    Также способность силы Лоренса связывать механическое смещение с электрическим током представляет большой интерес для медицинской акустики.

    Рекомендованная литература и полезные ссылки

    • Болотовский Б. М. Оливер Хевисайд. — Москва: Наука, 1985. — С. 43-44. — 260 с.
    • Матвеев А. Н. Механика и теория относительности. — 3-е изд. — М. Высшая школа 1976. — С. 132.

    Сила Лоренса, видео

    При написании статьи старался сделать ее максимально интересной, полезной и качественной. Буду благодарен за любую обратную связь и конструктивную критику в виде комментариев к статье. Также Ваше пожелание/вопрос/предложение можете написать на мою почту [email protected] или в Фейсбук, с уважением автор.

    Источник: https://www.poznavayka.org/fizika/sila-lorentsa/

    Сила ? Лоренца — как действует и в чем ? измеряется? Как определить силу Лоренца?

    Нигде еще школьный курс физики так сильно не перекликается с большой наукой, как в электродинамике. В частности, ее краеугольный камень – воздействие на заряженные частицы со стороны электромагнитного поля, нашло широкое применение в электротехнике.

    Формула силы Лоренца

    Формула описывает взаимосвязь магнитного поля и основных характеристик движущегося заряда. Но сперва нужно разобраться, что же оно собой представляет.

    Определение и формула силы Лоренца

    В школе очень часто показывают опыт с магнитом и железными опилками на бумажном листе. Если расположить его под бумагой и слегка потрясти, то опилки выстроятся по линиям, которые принято называть линиями магнитной напряженности.

    Говоря простыми словами, это силовое поле магнита, которое окружает его подобно кокону. Оно замкнуто само на себя, то есть не имеет ни начала, ни конца. Это векторная величина, которая направлена от южного полюса магнита к северному.

    Если бы в него влетела заряженная частица, то поле воздействовало бы на него очень любопытным образом. Она бы не затормозилась и не ускорилась, а всего лишь отклонилась в сторону. Чем она быстрее и чем сильнее поле, тем больше на нее действует эта сила. Ее назвали силой Лоренца в честь ученого-физика, впервые открывшего это свойство магнитного поля.

    • Вычисляют ее по специальной формуле:
    • FЛ=qvB,
    • здесь q – величина заряда в Кулонах, v – скорость, с которой движется заряд, в м/с, а B – индукция магнитного поля в единице измерения Тл (Тесла).

    Направление силы Лоренца

    Ученые заметили, что есть определенная закономерность между тем, как частица влетает в магнитное поле и тем, куда оно ее отклоняет.

    Чтобы ее было легче запомнить, они разработали специальное мнемоническое правило. Для его запоминания нужно совсем немного усилий, ведь в нем используется то, что всегда под рукой – рука.

    Точнее, левая ладонь, в честь чего оно носит название правила левой руки.

    Итак, ладонь должна быть раскрыта, четыре пальца смотрят вперед, большой палец оттопырен в сторону. Угол между ними составляет 900.

    Теперь необходимо представить, что магнитный поток представляет собой стрелу, которая впивается в ладонь с внутренней стороны и выходит с тыльной.

    Пальцы при этом смотрят туда же, куда летит воображаемая частица. В таком случае большой палец покажет, куда она отклонится.

    Интересно!

    Важно отметить, что правило левой руки действует только для частиц со знаком «плюс». Чтобы узнать, куда отклонится отрицательный заряд, нужно четыре пальца направить в сторону, откуда летит частица. Все остальные манипуляции остаются прежними.

    Следствия свойств силы Лоренца

    Тело влетает в магнитном поле под каким-то определённым углом.

    Интуитивно понятно, что его величина имеет какое-то значение на характер воздействия на него поля, здесь нужно математическое выражение, чтобы стало понятнее.

    Следует знать, что как сила, так и скорость являются векторными величинами, то есть имеют направление. То же самое относится и к линиям магнитной напряженности. Тогда формулу можно записать следующим образом:

    FЛ=qvBsinα,

    sin α здесь – это угол между двумя векторными величинами: скоростью и потоком магнитного поля.

    Как известно, синус нулевого угла также равен нулю. Получается, что если траектория движения частицы проходит вдоль силовых линий магнитного поля, то она никуда не отклоняется.

    В однородном магнитном поле силовые линии имеют одинаковое и постоянное расстояние друг от друга. Теперь представим, что в таком поле перпендикулярно этим линиям движется частица. В этом случае сила Лоуренса заставит двигаться ее по окружности в плоскости, перпендикулярной силовым линиям. Чтобы найти радиус этой окружности, нужно знать массу частицы:

    R=mvqB

    Значение заряда не случайно взято как модуль. Это означает, что неважно, отрицательная или положительная частица входит в магнитное поле: радиус кривизны будет одинаков. Изменится только направление, в котором она полетит.

    1. Во всех остальных случаях, когда заряд имеет определенный угол α с магнитным полем, он будет двигаться по траектории, напоминающей спираль с постоянным радиусом R и шагом h. Его можно найти по формуле:
    2. R=mvsinαqB
    3. h=2mvcosαqB

    Еще одним следствием свойств этого явления является тот факт, что она не совершает никакой работы. То есть она не отдает и не забирает энергию у частицы, а лишь меняет направление ее движения.

    Самая яркая иллюстрация этого эффекта взаимодействия магнитного поля и заряженных частиц – это северное сияние. Магнитное поле, окружающее нашу планету, отклоняет заряженные частицы, прилетающие от Солнца. Но так как оно слабее всего на магнитных полюсах Земли, то туда проникают электрически заряженные частицы, вызывая свечение атмосферы.

    Центростремительное ускорение, которое придается частицам, используется в электрических машинах – электродвигателях. Хотя уместнее здесь говорить о силе Ампера – частном проявлении силы Лоуренса, которая воздействует на проводник.

    Принцип действия ускорителей элементарных частиц также основан на этом свойстве электромагнитного поля. Сверхпроводящие электромагниты отклоняют частицы от прямолинейного движения, заставляя их двигаться по кругу.

    Самое любопытное заключается в том, что сила Лоренца не подчиняется третьему закону Ньютона, который гласит, что всякому действию есть свое противодействие.

    Связано это с тем, что Исаак Ньютон верил, что всякое взаимодействие на любом расстоянии происходит мгновенно, однако это не так. На самом деле оно происходит с помощью полей.

    К счастью, конфуза удалось избежать, так как физикам удалось переработать третий закон в закон сохранения импульса, который выполняется в том числе и для эффекта Лоуренса.

    Формула силы Лоренца при наличии магнитного и электрического полей

    Магнитное поле имеется не только у постоянных магнитов, но и у любого проводника электричества. Только в данном случае помимо магнитной составляющей, в ней присутствует еще и электрическая. Однако даже в этом электромагнитном поле эффект Лоуренса продолжает свое воздействие и определяется по формуле:

    FЛ=qE+vB

    где v – скорость электрически заряженной частицы, q – ее заряд, B и E – напряженности магнитного и электрических полей поля.

    Единицы измерения силы Лоренца

    Как и большинство других физических величин, которые действуют на тело и изменяют его состояние, она измеряется в ньютонах и обозначается буквой Н.

    Понятие напряженности электрического поля

    Электромагнитное поле на самом деле состоит из двух половин – электрической и магнитной. Они точно близнецы, у которых все одинаково, но вот характер разный. А если приглядеться, то во внешности можно заметить небольшие различия.

    То же самое касается и силовых полей. Электрическое поле тоже обладает напряженностью – векторной величиной, которая является силовой характеристикой.

    Она воздействует на частицы, которые в неподвижности находятся в нем.

    Само по себе оно не является силой Лоренца, ее просто нужно принимать во внимание, когда вычисляется воздействие на частицу в условиях наличия электрического и магнитного полей.

    Напряженность электрического поля

    • Напряженность электрического поля воздействует только на неподвижный заряд и определяется по формуле:
    • E=Fq
    • Единицей измерения является Н/Кл или В/м.

    Примеры задачи

    Задача 1

    На заряд в 0,005 Кл, который движется в магнитном поле с индукцией 0,3 Тл, действует сила Лоренца. Вычислить ее, если скорость заряда 200 м/с, а движется он под углом 450 к линиям магнитной индукции.

    Дано:
    1. q = 0,005 Кл
    2. B = 0,3 Тл
    3. v = 200 м/с
    4. α = 450
    Решение:
    В условиях задачи нет упоминания электрического поля, поэтому силу Лоренца можно найти по следующей формуле:
    FЛ=qvBsinα=0,005×200×0,3×sin 450 =0,3×22=0,21 Н

    Задача 2

    Определить скорость тела, имеющего заряд и которое движется в магнитном поле с индукцией 2 Тл под углом 900. Величина, с которой поле воздействует на тело, равна 32 Н, заряд тела – 5 × 10-3 Кл.

    Дано:
    • q = 0,005 Кл
    • B = 2 Тл
    • FЛ = 32 Н
    • α = 900
    Решение:
    1. Чтобы найти скорость заряда, необходимо несколько видоизменить формулу для нахождения силы Лоренца:
    2. FЛ=qvBsinαv=FЛqBsinα
    3. v=320,005×2×sin900=320,01×1=32000мс=32 км/с

    Задача 3

    Электрон движется в однородном магнитном поле под углом 900 ее силовым линиям. Величина, с которой поле воздействует на электрон, равна 5 × 10-13 Н. Величина магнитной индукции равна 0,05 Тл. Определить ускорение электрона.

    Дано:
    • q = -1,6 × 10-19 Кл
    • B = 0,05 Тл
    • FЛ = 5 × 10-13 Н
    • α = 900
    Решение:
    1. В этой задаче сила Лоренца ко всему прочему еще и заставляет двигаться электрон по окружности. Поэтому здесь под ускорением следует понимать центростремительное ускорение:
    2. aц=v2R
    3. На данный момент неизвестны ни скорость электрона, ни радиус окружности, по которой он движется.
    4. v=FЛqBsinα=5×10-13-1,6×10-19×0,05∙sin900=6×107мс
    5. R=mvqB=9×10-31×6×107-1,6×10-19×0,05=6,8×10-3мс

    aц=v2R=6×10726,8×10-3=5×1017мс2

    Электродинамика оперирует такими понятиями, которым трудно подобрать аналогию в обычном мире. Но это совсем не значит, что их невозможно постичь. С помощью различных наглядных экспериментов и природных явлений процесс познания мира электричества может стать по настоящему захватывающим.

    Источник: https://remont220.ru/osnovy-elektrotehniki/883-sila-lorentsa-i-vse-pro-nee/

    Сила Лоренца: определение, формулы, правило левой руки

    В статье расскажем про магнитную силу Лоренца, как она действует на проводник, рассмотрим правило левой руки для силы Лоренца и момент силы действующий на контур с током.

    Сила Лоренца — это сила, которая действует на заряженную частицу, падающую с определенной скоростью в магнитное поле. Величина этой силы зависит от величины магнитной индукции магнитного поля B, электрического заряда частицы q и скорости v, с которой частица падает в поле.

    То, как магнитное поле B ведет себя по отношению к нагрузке полностью отличается от того, как это наблюдается для электрического поля Е.

     Прежде всего, поле B не реагирует на нагрузку.

     Однако когда нагрузка перемещается в поле B, появляется сила, которая выражается формулой, которую можно рассматривать как определение поля B:

    Таким образом, видно, что поле B выступает в качестве силы, перпендикулярной к направлению вектора скорости V нагрузок и направление вектора B. Это можно проиллюстрировать на диаграмме:

    На диаграмме q положительный заряд!

    Единицы поля B могут быть получены из уравнения Лоренца. Таким образом, в системе СИ единица B равна 1 тесла (1T). В системе CGS полевой единицей является Гаусс (1G). 1T = 104G

    Движение заряда в поле B показано на анимации

    Для сравнения показана анимация движения как положительного, так и отрицательного заряда.

    Когда поле B охватывает большую площадь, заряд q, движущийся перпендикулярно направлению вектора B, стабилизирует свое движение по круговой траектории. Однако, когда вектор v имеет компонент, параллельный вектору B, тогда путь заряда будет спиралью, как показано на анимации

    Сила Лоренца на проводник с током

    Сила, действующая на проводник с током, является результатом силы Лоренца, действующей на движущиеся носители заряда, электроны или ионы. Если в разделе направляющей длиной l, как на чертеже

    полный заряд Q движется, тогда сила F, действующая на этот сегмент, равна

    Частное Q / t является значением протекающего тока I и, следовательно, сила, действующая на участок с током, выражается формулой

    Чтобы учесть зависимость силы F от угла между вектором B и осью отрезка, длина отрезка l была задана характеристиками вектора.

    Только электроны движутся в металле под действием разности потенциалов; ионы металлов остаются неподвижными в кристаллической решетке. В растворах электролитов анионы и катионы подвижны.

    Правило левой руки сила Лоренца

    Правило левой руки сила Лоренца — определяющее направление и возврат вектора магнитной (электродинамической) энергии.

    Если левая рука расположена так, что линии магнитного поля направлены перпендикулярно внутренней поверхности руки (чтобы они проникали внутрь руки), а все пальцы — кроме большого пальца — указывают направление протекания положительного тока (движущаяся молекула), отклоненный большой палец указывает направление электродинамической силы, действующей на положительный электрический заряд, помещенный в это поле (для отрицательного заряда, сила будет противоположная).

    Второй способ определения направления электромагнитной силы заключается в расположении большого, указательного и среднего пальцев под прямым углом. При таком расположении указательный палец показывает направление линий магнитного поля, направление среднего пальца — направление движения тока, а также направление большого пальца силы. 

    Момент силы, действующий на контур с током в магнитном поле

    Момент силы, действующей на контур с током в магнитном поле (например, на проволочную катушку в обмотке электродвигателя), также определяется силой Лоренца. Если петля (отмеченная на схеме красным цветом) может вращаться вокруг оси, перпендикулярной полю B, и проводит ток I, то появляются две неуравновешенные силы F, действующие в стороны от рамы, параллельной оси вращения.

    Момент этих сил М

    Определим вектор магнитного момента контура

    Теперь мы можем сохранить крутящий момент в виде

    Эти силы, действующие на элементы петли перпендикулярно оси вращения, направлены и взаимно компенсируются.

    Источник: https://meanders.ru/sila-lorenca.shtml

    1.18. Сила Лоренца

    

    • Сила Ампера, действующая на отрезок проводника длиной Δl с силой тока I, находящийся в магнитном поле B, может быть выражена через силы, действующие на отдельные носители заряда.
    • Пусть концентрация носителей свободного заряда в проводнике есть n, а q – заряд носителя. Тогда произведение n q υ S, где υ – модуль скорости упорядоченного движения носителей по проводнику, а S – площадь поперечного сечения проводника, равно току, текущему по проводнику:
    • Выражение для силы Ампера можно записать в виде:
    • Так как полное число N носителей свободного заряда в проводнике длиной Δl и сечением S равно n S Δl, то сила, действующая на одну заряженную частицу, равна

    Эту силу называют силой Лоренца. Угол α в этом выражении равен углу между скоростью и вектором магнитной индукции  Направление силы Лоренца, действующей на положительно заряженную частицу, так же, как и направление силы Ампера, может быть найдено по правилу левой руки или по правилу буравчика. Взаимное расположение векторов , и для положительно заряженной частицы показано на рис. 1.18.1.

    Рисунок 1.18.1.Взаимное расположение векторов , и Модуль силы Лоренца численно равен площади параллелограмма, построенного на векторах и помноженной на заряд q

    Сила Лоренца направлена перпендикулярно векторам и

    При движении заряженной частицы в магнитном поле сила Лоренца работы не совершает. Поэтому модуль вектора скорости при движении частицы не изменяется.

    Если заряженная частица движется в однородном магнитном поле под действием силы Лоренца, а ее скорость лежит в плоскости, перпендикулярной вектору то частица будет двигаться по окружности радиуса

    Сила Лоренца в этом случае играет роль центростремительной силы (рис. 1.18.2).

    Рисунок 1.18.2.Круговое движение заряженной частицы в однородном магнитном поле

    Период обращения частицы в однородном магнитном поле равен

    Это выражение показывает, что для заряженных частиц заданной массы m период обращения не зависит от скорости υ и радиуса траектории R.

    Угловая скорость движения заряженной частицы по круговой траектории называется циклотронной частотой. Циклотронная частота не зависит от скорости (следовательно, и от кинетической энергии) частицы. Это обстоятельство используется в циклотронах – ускорителях тяжелых частиц (протонов, ионов). Принципиальная схема циклотрона приведена на рис. 1.18.3.

    Рисунок 1.18.3.Движение заряженных частиц в вакуумной камере циклотрона

    Между полюсами сильного электромагнита помещается вакуумная камера, в которой находятся два электрода в виде полых металлических полуцилиндров (дуантов). К дуантам приложено переменное электрическое напряжение, частота которого равна циклотронной частоте.

    Заряженные частицы инжектируются в центре вакуумной камеры. Частицы ускоряются электрическим полем в промежутке между дуантами. Внутри дуантов частицы движутся под действием силы Лоренца по полуокружностям, радиус которых растет по мере увеличения энергии частиц.

    Каждый раз, когда частица пролетает через зазор между дуантами, она ускоряется электрическим полем. Таким образом, в циклотроне, как и во всех других ускорителях, заряженная частица ускоряется электрическим полем, а удерживается на траектории магнитным полем.

    Циклотроны позволяют ускорять протоны до энергии порядка 20 МэВ.

    Однородные магнитные поля используются во многих приборах и, в частности, в масс-спектрометрах – устройствах, с помощью которых можно измерять массы заряженных частиц – ионов или ядер различных атомов.

    Масс-спектрометры используются для разделения изотопов, то есть ядер атомов с одинаковым зарядом, но разными массами (например, 20Ne и 22Ne). Простейший масс-спектрометр показан на рис. 1.18.4.

    Ионы, вылетающие из источника S, проходят через несколько небольших отверстий, формирующих узкий пучок. Затем они попадают в селектор скоростей, в котором частицы движутся в скрещенных однородных электрическом и магнитном полях.

    Электрическое поле создается между пластинами плоского конденсатора, магнитное поле – в зазоре между полюсами электромагнита. Начальная скорость заряженных частиц направлена перпендикулярно векторам и

    На частицу, движущуюся в скрещенных электрическом и магнитном полях, действуют электрическая сила и магнитная сила Лоренца. При условии E = υB эти силы точно уравновешивают друг друга.

    Если это условие выполняется, частица будет двигаться равномерно и прямолинейно и, пролетев через конденсатор, пройдет через отверстие в экране.

    При заданных значениях электрического и магнитного полей селектор выделит частицы, движущиеся со скоростью υ = E / B.

    Далее частицы с одним и тем же значением скорости попадают в камеру масс-спектрометра, в которой создано однородное магнитное поле Частицы движутся в камере в плоскости, перпендикулярной магнитному полю, под действием силы Лоренца.

    Траектории частиц представляют собой окружности радиусов R = mυ / qB’. Измеряя радиусы траекторий при известных значениях υ и B’ можно определить отношение q / m.

    В случае изотопов (q1 = q2) масс-спектрометр позволяет разделить частицы с разными массами.

    Современные масс-спектрометры позволяют измерять массы заряженных частиц с точностью выше 10–4.

    Рисунок 1.18.4.Селектор скоростей и масс-спектрометр

    Если скорость частицы имеет составляющую вдоль направления магнитного поля, то такая частица будет двигаться в однородном магнитном поле по спирали. При этом радиус спирали R зависит от модуля перпендикулярной магнитному полю составляющей υ┴ вектора а шаг спирали p – от модуля продольной составляющей υ|| (рис. 1.18.5).

    Рисунок 1.18.5.Движение заряженной частицы по спирали в однородном магнитном поле

    Таким образом, траектория заряженной частицы как бы навивается на линии магнитной индукции. Это явление используется в технике для магнитной термоизоляции высокотемпературной плазмы, то есть полностью ионизированного газа при температуре порядка 106 K.

    Вещество в таком состоянии получают в установках типа «Токамак» при изучении управляемых термоядерных реакций. Плазма не должна соприкасаться со стенками камеры. Термоизоляция достигается путем создания магнитного поля специальной конфиругации. В качестве примера на рис. 1.18.

    6 изображена траектория движения заряженной частицы в магнитной «бутылке» (или ловушке).

    Рисунок 1.18.6.Магнитная «бутылка». Заряженные частицы не выходят за пределы «бутылки». Магнитное поле «бутылки» может быть создано с помощью двух круглых катушек с током

    Аналогичное явление происходит в магнитном поле Земли, которое является защитой для всего живого от потоков заряженных частиц из космического пространства.

    Быстрые заряженные частицы из космоса (главным образом от Солнца) «захватываются» магнитным полем Земли и образуют так называемые радиационные пояса (рис. 1.18.

    7), в которых частицы, как в магнитных ловушках, перемещаются туда и обратно по спиралеобразным траекториям между северным и южным магнитными полюсами за времена порядка долей секунды. Лишь в полярных областях некоторая часть частиц вторгается в верхние слои атмосферы, вызывая полярные сияния.

    Радиационные пояса Земли простираются от расстояний порядка 500 км до десятков земных радиусов. Следует вспомнить, что южный магнитный полюс Земли находится вблизи северного географического полюса (на северо-западе Гренландии). Природа земного магнетизма до сих пор не изучена.

    Рисунок 1.18.7.Радиационные пояса Земли. Быстрые заряженные частицы от Солнца (в основном электроны и протоны) попадают в магнитные ловушки радиационных поясов. Частицы могут покидать пояса в полярных областях и вторгаться в верхние слои атмосферы, вызывая полярные сияния
    Модель. Движение заряда в магнитном поле
    Модель. Масс-спектрометр
    Модель. Селектор скоростей

     

    Лучшие школы, лагеря, ВУЗы за рубежом
    Гидра сайт
    сайт как можно быстрее и дешевле! Авторские фишки
    hydra2gate.com

    Источник: https://physics.ru/courses/op25part2/content/chapter1/section/paragraph28/theory.html

    Формула силы лоренца

    ОПРЕДЕЛЕНИЕ

    Сила Лоренца – сила, действующая на точечную заряженную частицу, движущуюся в магнитном поле.

    Она равна произведению заряда, модуля скорости частицы, модуля вектора индукции магнитного поля и синуса угла между вектором магнитного поля и скоростью движения частицы.

    Здесь – сила Лоренца, – заряд частицы, – модуль вектора индукции магнитного поля, – скорость частицы, – угол между вектором индукции магнитного поля и направления движения.

    Единица измерения силы – Н (ньютон) .

    Сила Лоренца — векторная величина. Сила Лоренца принимает своё наибольшее значение когда векторы индукции и направления скорости частицы перпендикулярны ().

    Направление силы Лоренца определяют по правилу левой руки:

    Если вектор магнитной индукции входит в ладонь левой руки и четыре пальца вытянуты в сторону направления вектора движения тока, тогда отогнутый в сторону большой палец показывает направление силы Лоренца.

    В однородном магнитном поле частица будет двигаться по окружности, при этом сила Лоренца будет центростремительной силой. Работа при этом не будет совершаться.

    Примеры решения задач по теме «Сила Лоренца»

    ПРИМЕР 1

    ПРИМЕР 2

    ЗаданиеПод действием силы Лоренца частица массы m с зарядом q движется по окружности. Магнитное поле однородно, его напряжённость равна B. Найти центростремительное ускорение частицы.

    РешениеВспомним формулу силы Лоренца:

    Кроме того, по 2 закону Ньютона:

    В данном случае сила Лоренца направлена к центру окружности и ускорение, ею создаваемое, направлено туда же, то есть это и есть центростремительное ускорение. Значит:

    1. Вычислите силу Лоренца, действующую на протон, движущийся со скоростью 106 м/с в однородном магнитном поле с индукцией 0,3 Тл перпендикулярно линиям индукции.
    2. В однородном магнитном поле с индукцией 0,8 Тл на проводник с током 30 А, длина активной части которого 10 см, действует сила 1,5 Н. Под каким углом к вектору магнитной индукции размещен проводник?
    3. Какие из частиц электронного пучка
    отклоняются на больший угол в одном и том же магнитном поле – быстрые или медленные? (почему?)
    4. Ускоренный в электрическом поле разностью потенциалов 1,5 105 В протон влетает в однородное магнитное поле перпендикулярно к линиям магнитной индукции и движется равномерно по окружности радиусом 0,6 м. Определите скорость протона, модуль вектора магнитной индукции и силу, с которой магнитное поле действует на протон.

    Литература: —

    Интернет ресурсы.

    Тема № 10 Электромагнитные колебания.

    Решение задач и упражнений по образцу.

    Прочтите теоретический материал, выбрав один из источников, указанных в списке литературы.

    Найти формулы для решения задач.

    Записать «Дано» к условию задачи.

    Задача 1. В колебательном контуре индуктивность катушки равна 0,2 Гн. Амплитуда силы тока 40 мА. Найдите энергию магнитного поля катушки и энергию электрического поля конденсатора в тот момент, когда мгновенное значение силы тока в 2 раза меньше амплитудного. Сопротивлением контура пренебречь.

    Задача 2. Рамка площадью 400 см 2 имеет 100 витков. Она вращается в однородном магнитном поле с индукцией 0,01 Тл, причём период вращения рамки равен 0,1с. Написать зависимость ЭДС от времени, возникающей в рамке, если ось вращения перпендикулярна к линиям магнитной индукции.

    Задача 3.На первичную обмотку трансформатора подаётся напряжение220В. Какое напряжение можно снять со вторичной обмотки этого трансформатора, если коэффициент трансформации равен 10? Будет ли он потреблять энергию из сети, если его вторичная обмотка разомкнута?

    Литература: — Г.Я. Мякишев Б.Б. Буховцев Физика. Учебник для 11 кл. – М., 2014.

    Интернет ресурсы.

    Ландсберг Г.С. Элементарный учебник физики – М. Высшая школа 1975.

    Яворский Б.М. Селезнев Ю.А. Справочное руководство по физике – М.Наука, 1984.

    Решение задач на расчет параметров колебательного контура.

    Прочтите теоретический материал, выбрав один из источников, указанных в списке литературы.

    Найти формулы для решения задач.

    Записать «Дано» к условию задачи.

    1. Какую необходимо взять емкость в колебательном контуре, чтобы при индуктивности 250 мГн можно было бы его настроить на звуковую частоту 500 Гц.

    2. Найти индуктивность катушки, если амплитуда напряжения равна 160 В, амплитуда силы тока 10 А, а частота 50 Гц.

    3. Конденсатор включен в цепь переменного тока стандартной частоты с напряжением 220В. Какова ёмкость конденсатора, если сила тока в цепи 2,5 А.

    4. В одном ящике находится резистор, в другом конденсатор, в третьем – катушка индуктивности. Выводы подключены к наружным зажимам. Как, не открывая ящиков, узнать, что находится в каждом из них? (Даются источники постоянного и переменного напряжения одинаковой величины и лампочка.)

    Литература: — Г.Я. Мякишев Б.Б. Буховцев Физика. Учебник для 11 кл. – М., 2014.

    Интернет ресурсы.

    Ландсберг Г.С. Элементарный учебник физики – М. Высшая школа 1975.

    Яворский Б.М. Селезнев Ю.А. Справочное руководство по физике – М.Наука, 1984.

    Нидерландский физик X. А. Лоренц в конце XIX в. установил, что сила, действующая со стороны магнитного поля на движущуюся заряженную частицу, всегда перпендикулярна направле­нию движения частицы и силовым линиям магнитного поля, в котором эта частица движется. Направление силы Лоренца можно определить с помощью правила левой руки. Если расположить ладонь левой руки так, чтобы четыре вытянутых пальца указывали на­правление движения заряда, а вектор магнитной индукции поля входил в отставленный большой палец укажет направление силы Лоренца, действующей на положительный заряд.

    Если заряд частицы отрицательный, то сила Лоренца будет направлена в противоположную сторону.

    Модуль силы Лоренца легко определяется из закона Ампера и составляет:

    F = | q | vB sin? ,

    где q — заряд частицы, v — скорость ее движения , ? — угол между векторами скорости и индукции магнитного поли.

    Если кроме магнитного поля есть еще и электрическое поле , которое действует на заряд с силой , то полная сила, действующая на заряд, равна:

    .

    Часто именно эту силу называют силой Лоренца, а силу, выраженную формулой (F = | q | vB sin? ) называют магнитной частью силы Лоренца .

    Поскольку сила Лоренца перпендикулярна направлению движения частицы, она не может изменить ее скорость (она не совершает работы), а может изменить лишь направление ее движения, т. е. искривить траекторию .

    Такое искривление траектории электронов в кинескопе телевизо­ра легко наблюдать, если поднести к его экрану постоянный магнит — изображение исказится.

    Движение заряженной частицы в однородном магнитном поле. Пусть заряженная частица влетает со скоростью v в однородное магнитное поле перпендикулярно линиям напряженности.

    Сила, действующая со стороны магнитного поля на частицу, заставит ее равномерно вращаться по окружности радиусом r , который легко найти, воспользовавшись вторым законом Ньютона , выражением целеустремленного ускорения и формулой (F = | q | vB sin? ):

    .

    Отсюда получим

    .

    где m — масса частицы.

    Применение силы Лоренца.

    Действие магнитного поля на дви­жущиеся заряды применяется, например, в масс-спектрографах , позволяющих разделять заряженные частицы по их удельным за­рядам, т. е. по отношению заряда частицы к ее массе, и по полу­ченным результатам точно определять массы частиц.

    Вакуумная камера прибора помещена в поле (вектор индукции перпендикулярен рисунку). Ускоренные электрическим полем заряженные частицы (электроны или ионы), описав дугу, попада­ют на фотопластину, где оставляют след, позволяющий с большой точностью измерить радиус траектории r . По этому радиусу опре­деляется удельный заряд иона. Зная заряд иона, легко вычислите его массу.

    • движение заряженной частицы в однородном магнитном поле;
    • применение силы Лоренца.
    В зависимости от планирования материала на изучение этой темы можно отвести от 1 до 3 уроков, включая уроки решения задач.

    Цели урока

    Изучить движение заряженной частицы в однородном магнитном поле, отработать решение задач по теме «Действие магнитного поля на движущийся заряд. Сила Лоренца».

    Новый материал на данном уроке изучается в ходе одновременной работы учащихся с компьютерной моделью. Ответы на вопросы рабочего листа учащиеся должны получить, используя возможности данной модели.

    № п/пЭтапы урокаВремя, минПриемы и методы
    1Организационный момент2
    2Повторение изученного материала по теме «Сила Лоренца»10Фронтальная беседа
    3Изучение нового материала с помощью компьютерной модели «Движение заряженной частицы в однородном магнитном поле»30Работа с рабочим листом и моделью
    4Объяснение домашнего задания3

    Домашнее задание: § 6, № 849 (Сб. задач. 10–11 кл. А. П. Рымкевич – Москва Дрофа, 2001).

    Рабочий лист к уроку

    Примерные ответы
    Модель «Движение заряда в магнитном поле»

    ФИО, класс __________________________________________________

    1.

    при каких условиях частица движется по окружности?

    Ответ: частица движется по окружности, если вектор скорости перпендикулярен вектору индукции магнитного поля.

    2.

    При условии, что частица двигается по окружности, выставьте максимальные значения скорости частицы и величины магнитной индукции поля. Чему равен радиус окружности, по которой движется частица?

    Ответ: R = 22,76 см.

    3.

    Уменьшите скорость частицы в 2 раза. Магнитное поле не меняйте. Чему равен радиус окружности, по которой движется частица?

    Ответ: R = 11,38 см.

    4.

    Уменьшите еще раз скорость частицы в 2 раза. Магнитное поле не меняйте. Чему равен радиус окружности, по которой движется частица?

    Ответ: R = 5,69 см.

    5.

    Как зависит радиус окружности, по которой движется частица от величины вектора скорости частицы?

    Ответ: радиус окружности, по которой движется частица, прямо пропорционален величине вектора скорости частицы.

    6.Вновь установите максимальные значения скорости и величины магнитной индукции поля (частица двигается по окружности).
    7.

    Уменьшите величину магнитной индукции в 2 раза. Скорость частицы не меняйте. Чему равен радиус окружности, по которой движется частица?

    Ответ: R = 45,51 см.

    8.

    Уменьшите величину магнитной индукции еще раз в 2 раза. Скорость частицы не меняйте. Чему равен радиус окружности, по которой движется частица?

    Ответ: R = 91,03 см.

    9.

    Как зависит радиус окружности, по которой движется частица от величины магнитной индукции поля?

    Ответ: радиус окружности, по которой движется частица, обратно пропорционален величине магнитной индукции поля.

    10.

    Используя формулу радиуса окружности, по которой движется заряженная частица в магнитном поле (в учебнике формула 1.6) вычислите удельный заряд частицы (отношение заряда частицы к его массе).


    11.

    Сравните удельный заряд частицы с удельным зарядом электрона. Сделайте вывод.

    Ответ: полученный результат соответствует табличному значению удельного заряда электрона.

    12.

    Пользуясь правилом левой руки, определите знак заряда частицы в компьютерном эксперименте. Сделайте вывод.

    Ответ: анализ траектории движения частицы по правилу левой руки позволяет сказать, что это отрицательно заряженная частица. Учитывая ранее полученный результат равенства удельных зарядов исследуемой частицы и электрона, можно сделать вывод о том, что частица, представленная в модели, является электроном.

    13.Следующие эксперименты выполните при данном условии: υ x = 5∙10 7 м/с, υ z = 0 м/с, B = 2 мТл.14.

    Вычислите силу Лоренца, действующую на заряд.


    15.

    Вычислите ускорение, которое сообщает этому заряду данная сила (по второму закону Ньютона).

    F Л = 1,6∙10 –14 Н,

    m = 9,1∙10 –31 кг.

    ____________________

    a – ?

    Ответ: ускорение заряда равно 1,76∙10 16 м/с 2 .

    16.

    Вычислите радиус окружности, по которой движется частица, используя формулу центростремительного ускорения.

    υ = 5∙10 7 м/с,

    a = 1,76∙10 16 м/с 2 .

    ____________________

    R – ?

    Похожие вопросы

    • Для молодших школярiв придбали всього 200 квиткiв: 74 квит.-в ляльковий театр. щосту частину решти-у цирк.а всi iншi- в кiнотеатр. Скiльки придбали в кiнотеатр,
    • спишите текст и продолжите его двумя-тремя предложениями. Жаркий летний день.В знойном воздухе разлита духота.Синее безоблачное небо подернуто легкой дымкой.
    • 1. Мяч упал с высоты 3м, отскочил от пола и был пойман на высоте 1м. Найти путь и перемещение мяча. 2. Скорость перемещения шагающего эскаватора во время работы равна 0,18 км/час. На какое расстояние передвинется эскаватор за 5 мин? 3. Расстояние между городами А и В рано 250 км. Одновременно из обоих городов навстречу друг другу выезжают две автомашины, одна со скоростью 60 км/час, другая 40 км/час. Через какое время они встретятся? 4. Движение материальной точки описывается уравнением x=-25+5t. Найти начальную координату точки величину и направление скорости, координату точки через 5 с. Начертите график зависимости координаты от времени. 5. Какое из тел не двигалось? Какое тело двигалось с меньшей скоростью? В одинаковом ли направлении двигались тела?
    • «Главные причины образования климата» Составьте схему.
    • Вместо многоточия необходимо вставить слово: 1) Believed to be an ancestor of domestic dog, the wolf is generated (1)… a highly intelligent animal. Wolves travel in packs and their territory can be anywhere (2)… 40 to 400 square miles. As well as marking the borders of their territory with scent, they (3)… other wolves know they are around by barking and howling. 2) A pack might (4)… of up to 30 wolves, although where (5)… food supply is limited there might only be six or seven animals in the pack. When hunting, they work together to chase an animal, block (6) … escape, and finally catch it. In (7) … way, they are (8) … to trap large animals, such as deer or moose. 3) If farm animals are available, they (9) … the wolves with an easy source of food. This, of course, brings then (10) … contact with humans. Poisoning and shooting have contributed (11)… the decline in wolf populations around the world. The red wolf is now almost extinct (12) … the wild, while the grey wolf has (13) … its habitat reduced to a few areas in Europe, North America and Asia. (14) … mani other large mammals, the wolf is increasingly (15) … threat from human activity.

    Действие магнитного поля на движущийся заряд. Сила Лоренца

    Смотреть онлайн

    Действие магнитного поля на движущийся заряд. Сила Лоренца:

    Презентация на тему Действие магнитного поля на движущийся заряд. Сила Лоренца к уроку по физике

    Презентация «Действие магнитного поля на движущийся заряд. Сила Лоренца» онлайн бесплатно на сайте электронных школьных учебников school-textbook.com

    2 слайд

    Сила Лоренца Модуль силы Лоренца. Направление силы Лоренца Правило левой руки Плоские траектории движения заряженных частиц в однородном магнитном поле Вопросы по теме.

    3 слайд

    Что называют линиями магнитной индукции? Закон Ампера? Правило левой руки для определения направления силы Ампера. В каких единица выражается магнитная индукция?

    4 слайд

    Сила Лоренца — сила, действующая на движущуюся заряженную частицу со стороны магнитного поля. Х.Лоренц (1853-1928)–голландский физик, основатель электронной теории строения вещества.

    5 слайд

    Модуль силы Лоренца: Уравнение для силы тока в проводнике: Сила Ампера:

    9 слайд

    Согласно второму закону Ньютона: Отсюда радиус:

    10 слайд

    Если кисть левой руки расположить так, что четыре вытянутых пальца указывают направление скорости положительного заряда, а вектор магнитной индукции входит в ладонь, то отогнутый на 90 градусов большой палец покажет направление силы действующей на данный заряд.

    11 слайд

    Плоские траектории движения заряженных частиц в однородном магнитном поле Заряженная частица влетающая в однородное магнитное поле параллельно линиям магнитной индукции, движется равномерно вдоль этих линий. Вращение отрицательного заряда по окружности происходит в направлении противоположенном вращению положительного заряда (рис.в)

    12 слайд

    № 1 Какая сила действует на протон, движущийся со скоростью 10 Мм/с в магнитном поле индукцией 0,2 Тл перпендикулярно линиям индукции? № 2 Протон в магнитном поле индукцией 0,01 Тл описал окружность радиусом 10 см. Найти скорость протона.

    13 слайд

    Каким образом, зная силу Ампера , можно найти силу Лоренца? Дайте определение силе Лоренца. Чему равен её модуль? Как определяется направление силы Лоренца с помощью правила левой руки? Почему заряженная частица, влетающая в однородное магнитное поле в плоскости, перпендикулярно линиями магнитной индукции, движется по окружности? В каком случае частица движется в магнитном поле прямолинейно? Докажите, что период обращения по окружности заряженной частицы в поперечном магнитном поле не зависит от её скорости.

    Отзывы на school-textbook.com «Действие магнитного поля на движущийся заряд. Сила Лоренца» (0)

    Оставить отзыв

    Что бы вы не тратили своё драгоценное время на просмотр фильма, который не понравился большой массе зрителей, мы создали рейтинг просмотра, по которому вы сами сможете решить смотреть вам данную картину или нет.

    Рейтинг оценивается по 10 бальной шкале. Верхняя часть рейтинга (большими буквами) определяет рейтинг по версии «Кинопоиск», а в нижней части рейтинг по версии сайта «IMDB»

    Пример: 8.45 — оценка, данные значения для каждой киноленты разные. (45767) — количество зрителей которые проголосовали за данный фильм.

    По мнению пользователей оценки можно распределить по следующей шкале:

    1.1-1.9 — ужаснее некуда, стыдно смотреть такое. 2.0-2.9 — ужас, не советую 3.0-3.9 — Не понравился большой части аудитории, смотреть не стоит так считают многие киноманы. 4.0-4.9 — Обычный фильм, как многие говорят, ничего нового, но все, же смотреть можно. 5.0-6.5 — Хороший фильм, можно посмотреть, большой части аудитории данная лента понравилась. 6.6-7,9 — Очень хороший фильм, стоит обязательно посмотреть. 8.0-10.0 — Шедевр, в обязательном порядке посмотрите, уж точно не пожелеете! Зачастую такие фильмы получают награды, и являются прорывом в киноиндустрии!

    29. Сила Лоренца

    Сила Лоренца – сила, действующая на точечную заряженную частицу, движущуюся в магнитном поле.

    Она равна произведению заряда, модуля скорости частицы, модуля вектора индукции магнитного поля и синуса угла между вектором магнитного поля и скоростью движения частицы.

    FЛ = q υ B sin α.

    Сила Лоренца — векторная величина. Сила Лоренца принимает своё наибольшее значение когда векторы индукции и направления скорости частицы перпендикулярны.

    Направление силы Лоренца определяют по правилу левой руки:

    Если вектор магнитной индукции входит в ладонь левой руки и четыре пальца вытянуты в сторону направления вектора движения тока, тогда отогнутый в сторону большой палец показывает направление силы Лоренца.

    В однородном магнитном поле частица будет двигаться по окружности, при этом сила Лоренца будет центростремительной силой. Работа при этом не будет совершаться.

    30. Сила Ампера

    На проводник с током, находящийся в магнитном поле, действует сила, равная

    F = I·L·B·sina

    I — сила тока в проводнике; B — модуль вектора индукции магнитного поля; L — длина проводника, находящегося в магнитном поле; a — угол между вектором магнитного поля и направлением тока в проводнике.

    Силу, действующую на проводник с током в магнитном поле, называют силой Ампера.

    Максимальная сила Ампера равна:

                                                                           F = I·L·B

    Ей соответствует a = 900.

    Направление силы Ампера определяется по правилу левой руки: если левую руку расположить так, чтобы перпендикулярная составляющая вектора магнитной индукции В входила в ладонь, а четыре вытянутых пальца были направлены по направлению тока, то отогнутый на 90 градусов большой палец покажет направление силы, действующей на отрезок проводника с током, то есть силы Ампера.

    31. Сила взаимодействия токов

    Закон Ампера  — закон взаимодействия электрических токов. Впервые был установлен Андре Мари Ампером в 1820 для постоянного тока. Из закона Ампера следует, что параллельные проводники с электрическими токами, текущими в одном направлении, притягиваются, а в противоположных — отталкиваются.

    Наиболее известным примером, иллюстрирующим силу Ампера, является следующая задача. В вакууме на расстоянии r{\displaystyle r} друг от друга расположены два бесконечных параллельных проводника, в которых в одном направлении текут токи I1{\displaystyle I_{1}} и I2 {\displaystyle I_{2}}. Требуется найти силу, действующую на единицу длины проводника.

    В соответствии с законом Био-Савара-Лапласа бесконечный проводник с током I1{\displaystyle I_{1}} в точке на расстоянии {\displaystyle r}r создаёт магнитное поле с индукцией{\displaystyle B_{1}®={\frac {\mu _{0}}{4\pi }}{\frac {2I_{1}}{r}},}

    где {\displaystyle \mu _{0}}μ0 – магнитная постоянная.

    Теперь по закону Ампера найдём силу, с которой первый проводник действует на второй:

    {\displaystyle d{\vec {F}}_{1-2}=I_{2}d{\vec {l}}\times {\vec {B}}_{1}(r).}По правилу буравчика, {\displaystyle d{\vec {F}}_{1-2}} направлена в сторону первого проводника (аналогично и для {\displaystyle d{\vec {F}}_{2-1}}, а значит, проводники притягиваются).

    Модуль данной силы ({\displaystyle r} — расстояние между проводниками):

    {\displaystyle dF_{1-2}={\frac {\mu _{0}}{4\pi }}{\frac {2I_{1}I_{2}}{r}}dl.}

    Интегрируем, учитывая только проводник единичной длины (пределы {\displaystyle l} от 0 до 1):

    {\displaystyle F_{1-2}={\frac {\mu _{0}}{4\pi }}{\frac {2I_{1}I_{2}}{r}}.}

    Сила Ампера. Вывод через силу Лоренца. Электрический ток. Магнитная индукция. Формула

    Мы уже ввели логику того, что на движущийся в магнитном поле заряд действует сила. И опять нами была введена эта сила — сила Лоренца. Но сила Лоренца — сила, действующая на единичный заряд (т.е. одинокое тело), а если таких тел много? Например, если в магнитное поле помещён проводник с током. Ток — это упорядоченное движение заряженных частиц, тогда, если поместить проводник с током в магнитное поле, на каждый из зарядов будет действовать сила Лоренца (рис. 1).

    Рис. 1. Суммарная сила Лоренца

    Если просуммировать все эти силы, мы получим общую силу, действующую на проводник с током. Назовём эту силу — силой Ампера. Ток в проводнике организуется электронами (одинаковыми зарядами), и будем считать, что скорость продольного движения у них всех одинакова. Тогда суммарную силу Лоренца запишем как:

    (1)

    Вспомним определение силы тока:

    (2)
    • где
      • — время прохождения заряда.

    Подставим (2) в (1):

    (3)

    Пусть длина проводника — 

    , считая, что электроны движутся равномерно, то , тогда:

    (4)

    Сила (4) и является силой Ампера. Для определения направления силы Ампера пользуются правилом левой руки для силы Ампера: ориентируем левую руку так, чтобы линии магнитной индукции входили в ладонь, четыре пальца по току, тогда противопоставленный палец показывает направление силы Ампера.

    В ряде задач не лишним будет использование соотношение для момента силы Ампера. Такие задачи чаще всего связаны с контуром (замкнутой кривой), помещённой в магнитное поле. Моментом сил называется произведение силы на плечо силы, тогда:

    (5)

    Вывод: в задачах сила Ампера вводится в очень ограниченной системе. Проводник с током должен быть помещён в магнитное поле. Только тогда и возникает эта сила (4). Ещё использование сопряжено со втором законом Ньютона и дальнейшими кинематическими характеристиками движения.

    Поделиться ссылкой:

    5.1: Силы на свободных зарядах и токах

    Уравнение силы Лоренца и введение в силу

    Уравнение силы Лоренца (1.2.1) полностью характеризует электромагнитные силы на неподвижных и движущихся зарядах. Несмотря на простоту этого уравнения, оно очень точное и необходимо для понимания всех электрических явлений, поскольку эти явления наблюдаются только в результате действия сил на заряды. Иногда эти силы приводят в движение двигатели или другие исполнительные механизмы, а иногда они управляют электронами через материалы, которые нагреваются, освещаются или претерпевают другие физические или химические изменения.Эти силы также управляют токами, необходимыми для всех электронных схем и устройств.

    Когда электромагнитные поля, а также расположение и движение свободных зарядов известны, вычисление сил, действующих на эти заряды, не вызывает затруднений и объясняется в разделах 5.1.2 и 5.1.3. Когда эти заряды и токи заключены в проводники, а не изолированы в вакууме, обычно можно использовать подходы, представленные в разделе 5.2. Наконец, когда интересующие нас заряды и движение зарядов связаны внутри неподвижных атомов или вращающихся заряженных частиц, выражения плотности силы Кельвина развиты в разделе 5.3 необходимо добавить. Проблема обычно выходит за рамки этого текста, когда создающие силу электромагнитные поля не заданы, а определяются теми же зарядами, на которые действуют силы (например, физика плазмы), и когда скорости являются релятивистскими.

    В простейшем случае используются силы, возникающие из-за известных электромагнитных полей, действующих на свободные заряды в вакууме. Этот случай можно рассматривать с помощью уравнения силы Лоренца (5.1.1) для вектора силы \ (\ overline {\ mathrm {f}} \), действующего на заряд q [кулонов]:

    \ [\ overline {\ mathrm {f}} = \ mathrm {q} \ left (\ overline {\ mathrm {E}} + \ overline {\ mathrm {v}} \ times \ mu _ {\ mathrm {o} } \ overline {\ mathrm {H}} \ right) \ quad [\ text {Newtons}] \ qquad \ qquad \ qquad \ text {(Уравнение силы Лоренца)} \]

    , где \ (\ overline {\ mathrm {E}} \) и \ (\ overline {\ mathrm {H}} \) — локальные электрические и магнитные поля, а \ (\ overline {\ mathrm {v}} \) вектор скорости заряда [мс -1 ].

    Электрические силы Лоренца на свободных электронах

    Электронно-лучевая трубка (ЭЛТ) , используемая для дисплеев в старых компьютерах и телевизорах, как показано на рисунке 5.1.1, представляет собой простой пример закона силы Лоренца (5.1.1). Электроны, термически возбужденные нагретым катодом при -В, вылетают с низкой энергией и ускоряются в вакууме с ускорением \ (\ overline {\ mathrm {a}} \) [мс -2 ] к заземленному аноду электрическим полем \ (\ overline {\ mathrm {E}} \ cong- \ hat {z} \ mathrm {V} / \ mathrm {s} \) между анодом и катодом 13 ; V и s — напряжение на трубке и расстояние между катодом и анодом соответственно.В электронике анод всегда имеет более положительный потенциал \ (\ Phi \), чем катод, по определению.

    13 Анод заземлен по соображениям безопасности; он находится на лицевой стороне трубки, где пользователи могут положить пальцы на другую сторону стеклянной лицевой панели. Кроме того, катод и анод иногда имеют такую ​​форму, что электрическое поле \ (\ overline {\ mathrm {E}} \), сила \ (\ overline {\ mathrm {f}} \) и ускорение \ (\ overline {\ mathrm {a}} \) — это функции от z, а не постоянные; я.е., \ (\ overline {\ mathrm E} \ neq- \ hat {z} V / D \).

    Рисунок \ (\ PageIndex {1} \): электронно-лучевая трубка.

    Ускорение \ (\ overline {\ mathrm {a}} \) регулируется законом Ньютона :

    \ [\ overline {\ mathrm {f}} = \ mathrm {m} \ overline {a} \ qquad \ qquad \ qquad \ text {(закон Ньютона)} \]

    где m — масса безудержно ускоряющейся частицы. Следовательно, ускорение a заряда электрона q = -e в электрическом поле E = V / s равно:

    \ [\ mathrm {a} = \ mathrm {f} / \ mathrm {m} = \ mathrm {qE} / \ mathrm {m} \ cong \ mathrm {eV} / \ mathrm {ms} \ quad \ left [ \ mathrm {ms} ^ {- 2} \ right] \]

    Последующую скорость \ (\ overline {\ mathrm {v}} \) и положение z частицы можно найти путем интегрирования ускорения \ (\ hat {z} a \):

    \ [\ overline {\ mathrm {v}} = \ int_ {0} ^ {\ mathrm {t}} \ mathrm {\ overline {a}} (\ mathrm {t}) \ mathrm {dt} = \ overline {\ mathrm {v}} _ {\ mathrm {o}} + \ hat {z} \ mathrm {at} \ quad \ left [\ mathrm {ms} ^ {- 1} \ right] \]

    \ [z = z_ {0} + \ hat {z} \ bullet \ int_ {0} ^ {t} {\ overline {v}} (t) dt = z_ {0} + \ hat {z} \ bullet \ bar {v} _ {0} t + at ^ {2} / 2 \ \ text {[m]} \]

    , где мы определили начальное положение и скорость электрона в момент t = 0 как z o и \ (\ overline {\ mathrm {v}} _ {\ mathrm {o}} \) соответственно.

    Увеличение w k кинетической энергии электрона равно накопленной работе, совершаемой над ним электрическим полем \ (\ overline {\ mathrm {E}} \). То есть увеличение кинетической энергии электрона является произведением постоянной силы f, действующей на него, и расстояния s, на которое электрон переместился в направлении \ (\ overline {\ mathrm {f}} \), испытывая это сила. Если s — расстояние между анодом и катодом, то:

    \ [\ mathrm {w} _ {\ mathrm {k}} = \ mathrm {fs} = (\ mathrm {eV} / \ mathrm {s}) \ mathrm {s} = \ mathrm {eV} \ \ text {[J]} \]

    Таким образом, кинетическая энергия, приобретаемая электроном при движении через разность потенциалов V, равна эВ Джоулям.{\ mathrm {D}} \ mathrm {e} \ mathrm {E} _ {\ mathrm {z}} \ mathrm {d} \ mathrm {z} = \ mathrm {eV} \]

    Типичные значения V в телевизионных ЭЛТ обычно меньше 50 кВ, чтобы свести к минимуму опасное рентгеновское излучение, возникающее при воздействии электронов на люминофор на лицевой панели ЭЛТ, которая часто сделана из свинцового стекла, поглощающего рентгеновское излучение.

    На рисунке 5.1.1 также показано, как изменяющиеся во времени боковые электрические поля \ (\ overline {\ mathrm {E}} _ {\ perp} (\ mathrm {t}) \) могут применяться отклоняющими пластинами для сканирования Электронный луч проходит через лицевую панель ЭЛТ и «раскрашивает» изображение, которое будет отображаться.При более высоких напряжениях в трубке V электроны движутся так быстро, что боковые электрические силы не успевают действовать, и вместо этого используется магнитное отклонение, потому что боковые магнитные силы увеличиваются со скоростью электронов v.

    Магнитные силы Лоренца на свободных зарядах

    Альтернативный метод бокового сканирования электронного луча в ЭЛТ использует магнитное отклонение, создаваемое катушками, которые создают магнитное поле, перпендикулярное электронному лучу, как показано на рисунке 5.1.2.Магнитная сила Лоренца на заряде q = -e (1,6021 × 10 -19 кулонов) легко определяется из (5.1.1) и равна:

    \ [\ overline {\ mathrm {f}} = — \ mathrm {e} \ overline {\ mathrm {v}} \ times \ mu _ {\ mathrm {o}} \ overline {\ mathrm {H}} \ \ текст {[N]} \]

    Таким образом, проиллюстрированный электронный луч ЭЛТ будет отклонен вверх, где магнитное поле \ (\ overline {\ mathrm {H}} \), создаваемое катушкой, направлено из бумаги; величина силы, действующей на каждый электрон, составляет evμ o H [Н].{2} / 2 \]

    Следовательно, скорость электронов v = (2 эВ / м) 0,5 , где m — масса электрона (9,107 × 10 -31 кг), и боковое отклонение увеличивается с увеличением напряжения трубки V, тогда как оно уменьшается, если электростатическое отклонение составляет вместо этого.

    Другой случай магнитного отклонения проиллюстрирован на рисунке 5.1.3, где свободный электрон, движущийся перпендикулярно магнитному полю \ (\ overline {\ mathrm {B}} \), испытывает силу \ (\ overline {\ mathrm {f}} \) ортогонален его вектору скорости \ (\ overline {\ mathrm {v}} \), поскольку \ (\ overline {\ mathrm {f}} = \ mathrm {q} \ overline {\ mathrm {v}} \ times \ mu _ {\ mathrm {o}} \ overline {\ mathrm {H}} \).{2} R = m_ {e} v \ omega_ {e} \]

    , где v = ω e R. Мы можем решить (5.1.9) для этой «электронной циклотронной частоты » ω e :

    \ [\ omega _ {\ mathrm {e}} = \ mathrm {e} \ mu _ {\ mathrm {o}} \ mathrm {H} / \ mathrm {m} _ {\ mathrm {e}} \ qquad \ qquad \ qquad \ text {(электронная циклотронная частота)} \]

    , который не зависит от v и энергии электрона, при условии, что электрон не является релятивистским. Таким образом, величины магнитных полей можно измерить, наблюдая частоту излучения свободных электронов ω e в интересующей области.

    Пример \ (\ PageIndex {B} \): Cyclotron Motion

    Каков радиус \ (r_e \) циклотронного движения для 100 э.в. свободный электрон в магнитосфере Земли 14 где B ≅ 10 -6 тесла? Каков радиус \ (r_p \) свободного протона с той же энергией? Массы электронов и протонов составляют ~ 9,1 · 10 -31 и 1,7 · 10 -27 кг соответственно.

    Решение

    Магнитная сила, действующая на заряженную частицу, равна qvμ o H = ma = mv 2 / r, где скорость v следует из (5.{-6} \\ [4pt] & \ cong 34 \ \ mathrm {m} \ end {align *} \]

    для электронов и ~ 2,5 км для протонов.

    14 Магнитосфера простирается от ионосферы до нескольких радиусов планет; столкновения частиц редки по сравнению с циклотронной частотой.

    Визуализация ионных токов в растворе на основе эффекта Лоренца с использованием правильных значений подвижности ионов

    Дж Магнитный резонанс. Авторская рукопись; доступно в PMC 1 июня 2011 г.

    Опубликован в окончательной редакции как:

    PMCID: PMC2874604

    NIHMSID: NIHMS185081

    Ranjith S.Wijesinghe

    1 Кафедра физики и астрономии, Государственный университет Болла, Манси, Индиана

    Брэдли Дж. Рот

    2 Кафедра физики, Оклендский университет, Рочестер, Мичиган

    1 Кафедра физики и астрономии , Государственный университет Болла, Манси, Индиана

    2 Отделение физики, Оклендский университет, Рочестер, Мичиган

    * Адресная корреспонденция Ранджиту С. Виджесингхе, отдел физики и астрономии Государственного университета Болла, Манси, IN 47306.ude.usb@ehgnisejiwsr; Факс: 1-765-285-5674 Окончательная отредактированная версия этой статьи издателем доступна на сайте J Magn Reson. См. Другие статьи в PMC, в которых цитируется опубликованная статья.

    Abstract

    Труонг и его коллеги недавно опубликовали статью, в которой описывается новый метод, называемый визуализацией на основе эффекта Лоренца (LEI), для обнаружения ионных токов в растворе. Их главная цель состояла в том, чтобы доказать, что сила Лоренца, действующая на ионы в присутствии статического магнитного поля, может использоваться в качестве механизма контраста для измерения нейронных токов с помощью магнитно-резонансной томографии.Однако они не смогли использовать правильные значения подвижностей ионов. В этом исследовании мы использовали правильные значения подвижности ионов и показали, что LEI нельзя использовать в качестве механизма контраста для прямого изображения нейронных токов.

    Ключевые слова: Визуализация эффекта Лоренца, ионные токи, МРТ

    Введение

    Труонг, Аврам и Сонг недавно опубликовали отчет «Визуализация ионных токов в растворе с помощью эффекта Лоренца» [1]. Их цель состояла в том, чтобы определить влияние магнитных сил на движущиеся ионы и оценить потенциальное использование сил Лоренца в качестве механизма контраста для визуализации нейронных токов с помощью магнитно-резонансной томографии.Truong et al. рассматривается ион с зарядом q и массой m, движущийся со скоростью v в ионном растворе, подверженном действию электрического поля E и магнитного поля B . Уравнение движения иона содержит три силы: электрическую силу q E , магнитную силу q v × B (вместе называемую силой Лоренца) и силу сопротивления -b v .

    В своих симуляциях они смоделировали движение ионов меди и сульфата (эти ионы использовались в их экспериментах).Их результаты показали, что ионы «следуют по траекториям, которые изогнуты вниз, как предсказывает направление силы Лоренца». Truong et al. пришли к выводу, что «ионные токи с длительностью и плотностью тока того же порядка величины, что и индуцированные нейроэлектрической активностью в нервных волокнах и в головном мозге, могут быть обнаружены». [1]

    Рассмотрим уравнение. 1 и пока пренебрегаем магнитным полем. Предполагая, что ион достиг конечной скорости (его ускорение равно нулю), мы находим, что v = E (q / b).Таким образом, коэффициент q / b играет роль подвижности иона u (скорость иона, деленная на напряженность электрического поля). Для иона меди Truong et al. пусть q = 3,204 × 10 −19 C и b = 2,5 × 10 −18 кг / с, подразумевая u = 0,128 C · с / кг, или 0,128 (м / с) / (В / м). Измеренные подвижности нескольких ионов приведены в таблице и составляют порядка 7 × 10 -8 (м / с) / (В / м).

    Таблица 1

    Величина измеренной подвижности ионов в воде *

    Ион Подвижность (м / с) / (В / м) Источник
    натрия 5.2 × 10 −8 [2]
    калий 7,6 × 10 −8 [2]
    хлор 7,9 × 10 −8 [2]
    медь 5,4 × 10 −8 [3]
    сульфат 8,0 × 10 −8 [3]

    Чтобы оценить важность магнитного часть сил Лоренца на движение ионов, мы можем сформировать безразмерное число R, взяв отношение магнитной силы (qvB) к электрической силе (qE), или R = vB / E.Если R порядка единицы, магнитные силы и электрические силы примерно равны. Если R намного меньше единицы, магнитные силы малы по сравнению с электрическими силами. Мы можем оценить v, используя подвижность, v = uE, так что электрическое поле сокращается в соотношении и R = uB. Если мы воспользуемся значениями Truong et al. Для подвижности 0,128 и магнитного поля 4 Тл, то R будет 0,51. Если вместо этого использовать подвижность иона натрия, 5,2 × 10 -8 , то R будет 0,21 × 10 -6 . Таким образом, Truong et al.Моделирование, похоже, переоценило влияние магнитной составляющей силы Лоренца более чем в миллион раз по сравнению с тем, что можно было бы ожидать при нервной проводимости. Силы Лоренца будет труднее обнаружить с помощью магнитно-резонансной томографии, чем Truong et al. предложить.

    Методы

    Мы решили дифференциальное уравнение, заданное уравнением. (1) численно с использованием метода Эйлера [4] на компьютерном кластере Беовульфа государственного университета Болла, который представляет собой 32-узловой компьютер с 64 2.Процессоры Xeon с тактовой частотой 8 ГГц. Мы использовали временные приращения 10 пс при вычислении производных переменных в дифференциальном уравнении. Время вычисления траекторий, показанных на рисунках, варьировалось от 0,5 часа до 4,5 часов с учетом начальных условий, используемых для траектории.

    Следуя их предположениям, мы также предположили, что ионы меди и сульфата первоначально находились в состоянии покоя вблизи положительного и отрицательного электродов соответственно. Мы также использовали те же параметры, которые они использовали для создания траекторий, указанных в них: q (Cu 2+ ) = 3.204 × 10 −19 C, q (SO 4 2− ) = −3,204 × 10 −19 C, м (Cu 2+ ) = 1.055 × 10 −25 кг, м (SO 4 2−) = 1,595 × 10 −25 кг, расстояние между двумя электродами, d = 10 см, абсолютное напряжение электродов, U = 5V, магнитное поле, B, = 4 Тл. и b = 2,5 × 10 −18 кг / с. Мы также предположили, что статическое магнитное поле направлено в z-направлении. показывает траектории ионов меди и сульфата, полученные нами в ходе нашего расчета.Этот рисунок очень похож на их статью. Если бы у нас был доступ к начальным положениям ионов, которые они использовали в своих симуляциях, мы могли бы точно воссоздать их фигуру. Однако, когда мы использовали правильные значения подвижности ионов меди и сульфат-ионов, перечисленные в, и другие параметры, перечисленные выше, с теми же начальными условиями, которые использовались для создания, мы получили траектории движения ионов меди и сульфата, как показано на. Этот рисунок показывает, что ионы меди и сульфата идут совершенно другим путем, чем тот, который заявлен Truong et al.Фактически, траектории почти точно следуют за линиями электрического поля вне сферической экспериментальной установки. Таким образом, этот расчет показывает, что основная сила, действующая на ионы, — это электрическая сила, создаваемая электродами в их экспериментальной установке. Влияние магнитной части силы Лоренца на ионы незначительно.

    Смоделированные траектории ионов Cu 2+ и SO 4 2- в сфере, подвергнутой воздействию однородного статического магнитного поля и дипольного электрического поля, индуцированного двумя электродами, расположенными на каждой стороне сферы, как было предложено Truong et al. al.Мы использовали те же параметры, что и при создании своей бумаги.

    Моделируемые траектории ионов Cu 2+ и SO 4 2- в сфере, подверженной воздействию однородного статического магнитного поля и дипольного электрического поля, индуцированного двумя электродами, расположенными с каждой стороны сферы, с правильными значениями для иона мобильность. Линии тока ограничены половиной плоскости, потому что мы использовали те же начальные условия, которые использовались при моделировании траекторий в.Если бы мы выбрали симметричное распределение начальных условий, линии тока располагались бы симметрично в верхней и нижней частях графика.

    Это исследование доказывает, что магнитная часть эффекта силы Лоренца не может использоваться в качестве механизма контраста для визуализации ионных токов во время метода магнитного резонанса.

    Другой механизм, который может влиять на сигнал MR, — это магнитогидродинамически индуцированный поток [5]. Проводящая жидкость в скрещенных магнитном и электрическом полях будет стремиться течь со скоростью порядка E / B = (50 В / м) / (4 Тл) = 12 м / с.Однако этот поток развивается за время, равное ρ / σB 2 , где ρ — плотность жидкости, а σ — проводимость. Truong et al. измеренное σ = 140 См / м, поэтому время составляет (1000 кг / м 3 ) / (140 См / м 16 T 2 ) = 0,4 с. Импульсы тока продолжаются всего около 30 мс, поэтому наведенные скорости, вероятно, были ближе к 1 м / с. Тем не менее, такой поток должен привести к значительному артефакту в МР-сигнале, и это может быть то, что Truong et al. наблюдаемый. Скотт и др. [6] наблюдали магнитогидродинамические эффекты при измерении тока методом MR.Такой эффект должен быть меньше в ткани из-за меньшей проводимости в 100 раз и из-за ограниченного потока воды в пористой ткани. Оба эффекта, исследованные Truong et al. и магнитогидродинамический эффект возникает из-за силы Лоренца, но в остальном это очень разные механизмы: магнитогидронамический поток зависит от плотности жидкости, проводимости (и, следовательно, концентрации ионов) и вызывается объемным потоком жидкости, тогда как сила Лоренца, проанализированная Truong et al.не зависит от этих факторов.

    Благодарность

    Это исследование было поддержано грантом R01EB008421 Национального института здравоохранения и Академией наук Индианы.

    Сноски

    Заявление издателя: Это PDF-файл неотредактированной рукописи, принятой к публикации. В качестве услуги для наших клиентов мы предоставляем эту раннюю версию рукописи. Рукопись будет подвергнута копированию, верстке и рассмотрению полученного доказательства, прежде чем она будет опубликована в окончательной форме для цитирования.Обратите внимание, что во время производственного процесса могут быть обнаружены ошибки, которые могут повлиять на содержание, и все юридические оговорки, относящиеся к журналу, имеют отношение.

    Ссылки

    1. Чыонг Т-К, Аврам А., Сонг А.В. Визуализация ионных токов в растворе с эффектом Лоренца. J. Magn. Резон. 2008; 191: 93–99. [PubMed] [Google Scholar] 2. Кац Б. Нерв, мышцы и синапс. Нью-Йорк: Макгроу Хилл; 1966. [Google Scholar] 3. Конвей БЫТЬ. Электрохимические данные. Вестпорт, CN: Greenwood Press; 1952 г. [Google Scholar] 4.Press QA, Flannery BP, Teukolsky SA, Vetterling WT. Численные рецепты: Искусство научных вычислений. Нью-Йорк: издательство Кембриджского университета; 1986. [Google Scholar] 5. Джексон JD. Классическая электродинамика. 2-е издание. Нью-Йорк: Джон Уайли и сыновья; 1975. [Google Scholar] 6. Скотт Г.К., Джой MLG, Армстронг Р.Л., Хенкельман Р.М. Измерение неоднородной плотности тока методом магнитного резонанса. IEEE Trans. на Мед. Визуализация. 1991. 10 (3): 362–374. [PubMed] [Google Scholar]

    Закон силы Лоренца | Изучение.com

    Электрическая сила

    Прежде чем мы перейдем к закону силы Лоренца, давайте отдельно рассмотрим электрические и магнитные силы. Начнем с электрической силы. Представьте, что у вас есть частица с некоторым зарядом ( q ). Например, это может быть отрицательно заряженный электрон или положительно заряженный протон.

    Теперь давайте посмотрим, что происходит, когда мы помещаем две заряженные частицы рядом друг с другом. Любой заряженный объект создает электрическое поле ( E ).Кроме того, любая заряженная частица, помещенная в электрическое поле, ощущает силу, создаваемую этим полем. Мы называем это электрической силой ( Fe ).

    Линии электрического поля от электрона.

    Вы, наверное, уже слышали, что одинаковые заряды отталкиваются, а противоположные — притягиваются. Эти объекты создают силы друг на друга через свои электрические поля, которые либо стягивают их вместе, либо раздвигают.

    Глядя на электрическое поле, действующее на одну заряженную частицу, мы обнаруживаем, что сила, которую оно создает на этой частице, равна заряду частицы, умноженному на электрическое поле.

    Что происходит, когда эта сила воздействует на нашу заряженную частицу? Одна вещь, которая может произойти, — это то, что он может начать двигаться. Оказывается, что-то действительно интересное происходит, когда заряженные частицы движутся.

    Магнитная сила

    Когда у вас есть сгусток заряженных частиц, движущихся в одном направлении, он создает электрический ток.Электрические токи в современной жизни есть повсюду. Например, в каждом электрическом устройстве, которое вы подключаете к стене, проходит ток заряженных электронов.

    Когда у вас есть два электрических тока, например, в электрических проводах, рядом друг с другом они передают силы друг на друга. Если токи текут в одном направлении, силы притягивают два тока, но если они текут в противоположных направлениях, они отталкивают друг друга.

    Магнитные поля, создаваемые током

    Происходит то, что движущиеся заряженные частицы создают магнитное поле ( B ).Это магнитное поле создается не только движущимися заряженными частицами, но и только движущиеся заряженные частицы ощущают создаваемую им силу. Ни на одну неподвижную заряженную частицу эта сила вообще не повлияет.

    Поскольку заряженная частица должна двигаться, чтобы почувствовать магнитную силу , имеет смысл привязать эту силу к скорости этой частицы ( v ). Глядя на формулу для магнитной силы ( Fb ), мы видим, что это действительно так.

    Магнитная сила равна заряду частицы, умноженному на произведение скорости и магнитного поля. Важно отметить, что «x» в уравнении очень конкретно является знаком произведения между двумя векторами, v и B , а не стандартным знаком умножения, который вы видите при умножении двух скаляров.

    Закон силы Лоренца

    Теперь, когда мы увидели магнитные и электрические силы, создаваемые соответствующими полями, мы, наконец, можем взглянуть на закон силы Лоренца.Когда движущаяся заряженная частица находится в присутствии магнитного и электрического поля, закон силы Лоренца говорит нам, что полная сила этих полей, действующая на заряженную частицу, равна сумме электрической и магнитной сил.

    Подставляя в наши уравнения электрических и магнитных сил, приведенные ранее, мы получаем стандартную формулу закона силы Лоренца.

    С помощью закона силы Лоренца мы берем наши отдельные электрические и магнитные силы и связываем их вместе в одну общую электромагнитную силу.

    Краткое содержание урока

    Все силы во Вселенной можно классифицировать по четырем фундаментальным силам природы. Например, все электрические и магнитные силы подпадают под категорию электромагнитной силы . Чтобы увидеть, как именно электрические и магнитные силы могут быть связаны друг с другом, давайте посмотрим на их соответствующие поля.

    Любая заряженная частица в электрическом поле будет чувствовать силу, создаваемую этим полем. Эта электрическая сила ( Fe ) равна заряду частицы ( q ), умноженному на электрическое поле ( E ).

    В отличие от электрических полей, неподвижная заряженная частица в магнитном поле не ощущает силы. Эта заряженная частица будет чувствовать силу, создаваемую магнитным полем, только если она имеет скорость; т.е. он должен двигаться. Эта магнитная сила ( Fb ) равна заряду частицы, умноженному на произведение скорости этой частицы ( v ) и магнитного поля ( B ).

    Эти две электрические и магнитные силы могут быть связаны вместе в одну электромагнитную силу посредством закона силы Лоренца . Этот закон гласит, что полная сила, действующая на заряженную частицу из-за электрического и магнитного полей, равна сумме электрических и магнитных сил, действующих на нее.

    Затем можно найти стандартную формулу закона силы Лоренца, записав наши уравнения электрических и магнитных сил в предыдущую формулу.

    Расходомер с силой Лоренца

    для жидкого алюминия: лабораторные эксперименты и заводские испытания

    Цель данной статьи — продемонстрировать, что поток расплавленного металла при высокой температуре может быть эффективно измерен бесконтактным способом с использованием внешних магнитных полей постоянного тока. Устройство, применяемое в настоящей работе, называется расходомером силы Лоренца (LFF) и основано на воздействии на поток магнитной системы и измерении силы сопротивления, действующей на нее.Сообщается о двух сериях измерений. В первой серии мы проводим модельный эксперимент в лаборатории с использованием эвтектического сплава GaInSn, который является жидким при комнатной температуре. Вторая серия измерений посвящена двум заводским испытаниям по измерению расхода жидкого алюминиевого сплава. В обоих тестах измеряется сила, действующая на магнитную систему, которая равна силе Лоренца, действующей на поток. Чтобы обобщить наши результаты, мы также выводим закон масштабирования, который связывает силу, действующую на локализованную магнитную систему, со скоростью потока жидкости с произвольной электропроводностью.Этот закон показывает, что LFF, если он правильно спроектирован, имеет широкий спектр потенциальных применений в черной и цветной металлургии.
    • Откуда: Металлургические операции и операции с материалами B, Том 42, выпуск 3, июнь 2011 г. (ASM-TMS-Springer)
    • Дата публикации: 1 июня 2011 г.
    • Страницы: 10

    Доступ для загрузки документа

    Для доступа приобретенный электронный документ (PDF), он появится в MyASM My Content. (Вы должны войти на сайт, чтобы получить доступ к приобретенному вами контенту).

    Вы также можете получить доступ к купленному документу, выполнив поиск и нажав кнопку «Загрузить» на странице сведений о продукте документа.

    Примечание : После загрузки вами цифрового контента ASM и оплаты любых применимых сборов ASM International предоставляет вам неисключительное право просматривать, использовать и отображать такой Контент неограниченное количество раз, исключительно на вашем персональном компьютере или персональном устройстве. и исключительно для вашего личного некоммерческого использования. Цифровой контент ASM предоставляется вам по лицензии, а не продается.Вы не можете продавать, сдавать в аренду, сдавать в аренду, распространять, транслировать, сублицензировать или иным образом передавать какие-либо права на Контент или любую его часть какой-либо третьей стороне, а также вы не можете удалять или изменять какие-либо уведомления о правах собственности или ярлыки на Контенте.

    Отрасли и области применения | Здания и инфраструктура

    Отрасли и приложения | Металлические изделия и оборудование

    Отрасли и области применения | Нанотехнологии

    Отрасли и приложения | Атомная энергетика

    Отрасли и области применения | Трубопроводы

    Обработка и обработка материалов | Пайка

    Обработка и обработка материалов | Порошковая металлургия

    Обработка и обработка материалов | Вторичная переработка

    Свойства и рабочие характеристики материалов | Коррозия

    Свойства и рабочие характеристики материалов | Электрические свойства

    Свойства и рабочие характеристики материалов | Трение и износ

    Испытания и оценка материалов | Механические испытания

    Испытания и оценка материалов | Неразрушающий контроль

    Металлы и сплавы | Нержавеющая сталь

    Неметаллические конструкционные материалы | Эластомеры

    Проблемы силы Лоренца и их решение

    1.Введение

    К сожалению, современная классическая электродинамика не лишена противоречий, которые до сих пор не получили своего объяснения.

    Основными уравнениями современной классической электродинамики являются уравнения Максвелла. Для вакуума они записываются следующим образом:

    (1,1)
    (1,2)
    (1,3)
    4)

    где и — напряженность электрического и магнитного поля, а — электрическая и магнитная индукция, — магнитная и диэлектрическая проницаемость вакуума. Из этих уравнений следуют волновые уравнения для электрического и магнитного поля

    (1,5)
    (1,6)

    эти уравнения показывают, что в вакууме могут быть расширены плоские электромагнитные волны, скорость распространения которых равна скорости свет

    (1.7)

    Уравнения Максвелла записаны для материальной среды

    (1,8)
    (1,9)
    (1,10)
    1 0247 9034 и

    9030 магнитные и диэлектрические постоянные среды, а также плотность, величина и скорость заряда.

    Уравнения (1.1 — 1.11) записаны в заданной инерциальной системе отсчета (IRS), и в них нет правил перехода одной IRS в другую. Данные уравнения также предполагают, что свойства заряда не зависят от их скорости, поскольку в первом члене правой части уравнения. (1.9) В качестве заряда принято его статическое значение.

    В уравнениях Максвелла не содержится указание на то, что является причиной силового взаимодействия токоведущих систем; поэтому необходимо ввести экспериментальный постулат о силе, действующей на движущийся заряд в магнитном поле.Этот постулат предполагает, что на заряд, движущийся в магнитном поле, действует сила

    (1.12)

    У этого подхода есть существенный недостаток. Если на движущийся заряд действует сила, то должен быть известен объект, со стороны которого действует эта сила. В этом случае магнитное поле является самостоятельным веществом, выступает в роли посредника между движущимися зарядами. Следовательно, не существует закона прямого действия, который дал бы ответ на вопрос, как взаимодействуют заряды, совершающие относительное движение.

    Отношение (1.12) вызывает недоумение. В механике силы, действующие на движущееся тело, связаны с его ускорением, при равномерном движении существуют силы трения. При равномерном движении существуют также силы трения. Направление этих сил совпадает с вектором скорости. Но сила, определяемая формулой. (1.12) обладают еще одним свойством. Прямолинейное движение вызывает силу, нормальную к направлению движения, что не допускается ни одним из существующих законов механики.Поэтому можно предположить, что это некий новый закон, касающийся относительного движения только заряженных тел.

    Несомненно, магнитное поле — одно из важных понятий современной электродинамики. В соответствии с законом Ампера вокруг тока, протекающего по проводнику, происходит циркуляция магнитного поля

    (1,13)

    где — ток проводимости. Если к току проводимости добавить ток смещения, то мы получим второе уравнение Максвелла (1.9).

    Следует отметить, что введение концепции магнитного поля не основано на какой-либо физической основе, но это констатация набора некоторых экспериментальных фактов, которые с помощью определенных математических процедур в больших количествах кейсы дают возможность получить правильный ответ при решении практических задач. Но есть ряд физических вопросов, на которые концепция магнитного поля не дает ответа. Используя уравнения.(1.12) и (1.13) нетрудно показать, что при однонаправленном параллельном движении двух одинаковых зарядов или потоков зарядов между ними должно возникать дополнительное притяжение. Однако если мы перейдем в инерциальную систему, которая движется вместе с зарядами, то там магнитное поле отсутствует, и нет никакого дополнительного притяжения. Этот парадокс в электродинамике не имеет объяснения. Не имеет объяснения и в специальной теории относительности (СТО).

    При силовом взаимодействии проводников, по которым протекает ток, силы действуют не только на движущиеся заряды, но и на решетку.Но понятие магнитного поля и на этот вопрос ответа не дает, т.к. В уравнениях. (1.1-1.13) наличие решетки не учитывается.

    Основным законом индукции в электродинамике считается Фарадейлов, следствием которого является первое уравнение Максвелла. Однако здесь есть проблемы. До сих пор считается, что униполярный генератор является исключением из правил потока. Существующее положение дел и те противоречия, которые с этим связаны, пожалуй, наиболее четко сформулированы в шестом томе работы [2].На странице 52 мы читаем: «правило потока утверждает, что ЭДС контура. равна скорости изменения магнитного потока через контур противоположного знака, когда поток изменяется либо с изменяющимся полем, либо из-за движения контура (или с тем и другим вместе). Два варианта — «смещение контура» или «изменение поля» неразличимы внутри правила. Тем не менее, мы используем эти два совершенно разных закона, чтобы объяснить правило для двух случаев: для «движущегося контура» и для «изменяющегося поля».И далее: в физике вряд ли найдется другой случай, когда простой и точный общий закон нужно интерпретировать в терминах двух разных явлений. Обычно такое красивое обобщение должно основываться на едином фундаментальном принципе. В нашем случае такой принцип отсутствует ».

    Все эти примеры свидетельствуют о том, что закон индукции Фарадея неточен или неполон и не отражает всех возможных версий возникновения электрического поля при изменении магнитного поля или при движении в Гер.

    Из вышесказанного можно сделать вывод, что физическая природа силы Лоренца, которая со времен Лоренца и Пуанкаре вводится аксиоматическим методом, нам пока неизвестна.

    2. Законы индукции

    Проведением экспериментов Фарадей установил, что в контуре индуцируется ток, когда в соседнем контуре включен или выключен постоянный ток или соседний контур с постоянным током движется относительно первого. контур.Поэтому в общем виде закон Фарадея записывается следующим образом [3]:

    (2.1)

    Эта запись закона указывает на то, что при определении циркуляции в подвижной системе координат рядом и должны стоять простые числа и должна приниматься полная производная. Но если циркуляция определяется в фиксированной системе координат, то штрихи рядом и отсутствуют, но в данном случае справа в уравнении. У (2.1) должна быть определенная производная по времени.

    Полная производная по времени в уравнении. (2.1) указывает на независимость конечного результата появления ЭДС в контуре от способа изменения потока. Поток может меняться как потому, что он зависит от времени, так и потому, что система, в которой определяется циркуляция, движется в магнитном поле, величина которого зависит от координат. Значение магнитного потока в уравнении. (2.1) определяется из уравнения

    (2.2)

    где магнитная индукция определяется в фиксированной системе координат, а элемент определяется в подвижной системе.Принимая во внимание уравнение. (2.1), получаем из уравнения. (2,16)

    Т.к., запишем:

    (2.3)

    В этом уравнении интеграл берется по контуру, который покрывает площадь. Сразу отметим, что все последующее изложение будет проводиться в предположении справедливости преобразований Галилея, т. Е. И. При движении в магнитостатическом поле выполняется уравнение

    Это уравнение получено не введением постулата о силе Лоренца, а непосредственно из закона индукции Фарадея.Таким образом, сила Лоренца является прямым следствием закона магнитоэлектрической индукции.

    Закон Фарадея (2.3) указывает, как изменение магнитного поля или движения в этих полях приводит к появлению электрического поля; поэтому его следует назвать законом магнитоэлектрической индукции. Однако в классической электродинамике нет закона индукции электрического магнето, который бы показал, как изменение электрического поля на них или движение в них приводит к появлению магнитного поля на них.Развитие классической электродинамики шло по другому пути. Впервые был известен закон Ампера

    .
    (2,4)

    где I — ток, который пересекает область, включенную в контур интегрирования. В дифференциальной форме Eq. (2.4) принимает вид:

    (2,5)

    где — плотность тока проводимости.

    Максвелл дополнил уравнение. (2.5) с током смещения

    (2,6)

    Однако должен существовать закон электромагнитной индукции, определяющий магнитные поля в изменяющемся электрическом поле

    (2.7)

    где — поток электрической индукции.

    (2,8)

    В отличие от магнитного поля, когда, для электрического поля и последнего члена в правой части уравнения.(2.8) он дает ток проводимости, и из соотношения (2.7) сразу следует закон Ампера. Таким образом, из уравнения. (2.7) следует закону Ампера. При движении в полях постоянного тока получаем

    (2,9)

    Как показано в работе [2], из уравнения. (2.9) следует и закону Био-Савара, если для подсчета магнитных полей взять электрические поля движущихся зарядов. В этом случае последний член правой части уравнения. (2.8) можно просто опустить, и законы индукции приобретут полностью симметричный вид

    (2.10)

    Для полей констант по законам преобразования они принимают следующий вид

    (2.11)

    3. Динамические потенциалы и поле движущихся зарядов

    Как уже упоминалось, в классической электродинамике отсутствует правило преобразования электрического и магнитного полей при передаче одного инерциальная система в другую. Этот недостаток устраняет SR. При всей математической обоснованности такого подхода физическая сущность таких преобразований до сих пор остается невыясненной.

    Поясним, какие потенциалы и поля могут генерировать движущиеся заряды. Первый шаг в этом направлении, продемонстрированный в работах [4,5,6,7], был сделан с помощью введения симметричных законов индукции (Ур. (2.10).

    Уравнения (2.10, 2.11) свидетельствуют о том, что в случае относительного движения системы отсчета между полями и возникает перекрестная связь, т. Е. Движение в полях приводит к появлению полей на и наоборот. . Движение в полях приводит к появлению полей и наоборот.Это приводит к дополнительным следствиям, которые впервые были рассмотрены в работе [4]. Электрическое поле за пределами длинного заряженного стержня определяется из уравнения

    где — линейный заряд.

    Если мы параллельно оси стержня в поле начнем двигаться со скоростью другого ИРС, то в нем появится дополнительное магнитное поле. Если мы теперь относительно уже движущейся ИСО начнем перемещать третью ИСО со скоростью, то уже за счет движения в поле появится добавка к электрическому полю.Этот процесс можно продолжать и дальше, в результате чего может быть получено число, которое дает значение электрического поля в движущемся IRS при достижении скорости, when, и. В конечном итоге в движущемся ИСО значение динамического электрического поля окажется больше, чем в исходном, и будет определяться уравнением:

    (3.1)

    Электрическое поле одиночного заряда будет определяться уравнением:

    где — нормальная составляющая скорости заряда вектора, который соединяет движущийся заряд и точку наблюдения.

    Уравнение для скалярного потенциала, создаваемого движущимся зарядом, для этого случая запишется следующим образом [4.5.6.7]:

    (3,2)

    где — скалярный потенциал фиксированного заряда. Потенциал можно назвать скалярно-векторным, поскольку он зависит не только от абсолютной величины заряда, но и от скорости и направления его движения относительно точки наблюдения. Максимальное значение этот потенциал имеет в направлении, перпендикулярном движению самого заряда.

    При движении в магнитном поле уже рассмотренным методом получаем:

    где — скорость, нормальная к направлению магнитного поля.

    Если применить полученные результаты к электромагнитной волне и обозначить компоненты потока на параллели скорости как ,, а, как нормальные к ней компоненты, то поля преобразования на них будут записаны:

    (3.3)

    где — импеданс свободного пространства, — скорость света.

    Поля преобразования Ур. (3.3) они были впервые получены в работе [4].

    4. Силовое взаимодействие параллельных проводников

    Понятие магнитного поля возникло в значительной степени из-за наблюдений силового взаимодействия токоведущих и намагниченных систем. Особенно важен опыт работы с железной стружкой, которую возводят возле полюсов магнита или вокруг кольцевого витка с током в четкие геометрические фигуры.Эти рисунки послужили поводом для введения такого понятия как силовые линии магнитного поля. В соответствии с третьим законом Ньютона при любом силовом взаимодействии всегда существует равенство действующих сил и противодействия, а также всегда есть те элементы системы, к которым эти силы приложены. Большим недостатком концепции магнитного поля является то, что оно не дает ответа на то, к чему конкретно прилагаются противодействующие силы, поскольку магнитное поле выступает как самостоятельная субстанция, с которой происходит взаимодействие движущихся зарядов.

    Экспериментально известно, что силы взаимодействия в токоведущих системах действуют на те проводники, движущиеся заряды которых создают магнитное поле. Однако в существующей концепции силового взаимодействия токоведущих систем, основанной на представлениях о магнитном поле и силе Лоренца, положительно заряженная решетка, которая является каркасом проводника и к которой приложены силы, не участвует в формирование сил взаимодействия.

    Разберем этот вопрос на основе концепции скалярно-векторного потенциала.Будем считать, что скалярно-векторный потенциал одиночного заряда определяется соотношением (3.2) и что электрические поля, создаваемые этим потенциалом, действуют на все окружающие заряды, в том числе на заряды положительно заряженной решетки.

    Рассмотрим с этих позиций силовое взаимодействие между двумя параллельными проводниками (рис. 1), по которым протекают токи. Мы будем считать, что и, представим положительные и отрицательные линейные заряды в верхнем и нижнем проводниках.

    Рис. ure 1. Схема силового взаимодействия токоведущих проводов двухпроводной схемы с учетом положительно заряженной решетки

    Мы также будем считать, что оба проводника до начала зарядов электрически нейтральны, т. Е. В проводниках есть две системы взаимно вставленных противоположных зарядов с линейными зарядами и, которые электрически нейтрализуют друг друга.На рисунке 1 эти системы разнесены по оси. Подсистемы с отрицательным зарядом (электроны) могут двигаться со скоростями,. Силу взаимодействия нижнего и верхнего проводников будем искать как сумму четырех сил, обозначение которых понятно из рисунка. Силы отталкивания и возьмем со знаком минус, а силы притяжения и со знаком плюс.

    Для одиночного участка двухпроводной цепи силы, действующей между отдельными подсистемами, будет записано

    (4.1)

    Сложив силы, получим сумму составной силы

    (4.2)

    где и берутся абсолютные значения линейных зарядов, а, принимают с его знаками.

    Где возьмем только два первых члена расширения в серии, т.е. будем учитывать это. Из уравнения. (4.2) получаем

    (4.3)

    Так как магнитное поле прямого провода, по которому течет ток, определяем по уравнению

    из уравнения.(4.3) получаем

    где — магнитное поле, создаваемое нижним проводником в месте расположения верхнего проводника.

    Аналог

    где — магнитное поле, создаваемое верхним проводником.

    Результаты, полученные в модели скалярно-векторного потенциала, полностью совпадают с результатами, полученными на основе концепции магнитного поля.

    Уравнение (4.3) представляет собой известное правило силового взаимодействия токоведущих систем, но получено его не феноменологическим путем на основе введения феноменологического магнитного поля, а на основе вполне понятных физических процедур, при предположение, что скалярный потенциал заряда зависит от скорости. В формировании сил взаимодействия в этом случае решетка принимает непосредственное участие, чего нет в модели магнитного поля.На рассматриваемой модели хорошо видны места приложения силы. Эти уравнения полностью совпадают с результатами, полученными на основе концепции магнитного поля. В этом случае берется только первый член расширения в серии. За скорости следует брать все условия расширения. Если учесть это обстоятельство, то связь между силами взаимодействия и скоростями заряда оказывается нелинейной. Это, в частности, приводит к тому, что закон силового взаимодействия токоведущих систем асимметричен.При одинаковых значениях токов, но при разных направлениях силы притяжения и отталкивания становятся неодинаковыми. Силы отталкивания оказываются больше силы притяжения. Эта разница небольшая и определяется уравнением

    но при скоростях носителей заряда близких к скорости света это может оказаться вполне ощутимым.

    Удалим решетку верхнего проводника и оставим только свободный электронный поток.В этом случае исчезнут силы,. В этом случае исчезнут силы и, что будет указывать на взаимодействие нижнего проводника с потоком свободных электронов, которые движутся со скоростью 10 мкс в месте расположения верхнего проводника. В этом случае значение силы взаимодействия определяется как:

    (4.4)

    Сила Лоренца предполагает линейную зависимость между силой, действующей на заряд, движущийся в магнитном поле, и его скоростью.Однако в полученном соотношении зависимость величины силы от скорости электронного потока будет нелинейной.

    Отметим еще один интересный результат. С учетом соотношения (4.3) силу взаимодействия электронного потока с прямолинейным проводником можно определить из уравнения

    (4,5)

    Из уравнения. Из (4.5) следует, что при одинаковом направлении движения электронов в проводнике и в электронном потоке при выполнении условий сила взаимодействия равна нулю.

    Поскольку скорость электронного потока обычно намного превышает скорость носителей тока в проводнике, вторым членом в скобках в соотношении (4.5) можно пренебречь. С

    получим магнитное поле, создаваемое нижним проводником в месте движения электронного потока:

    В этом случае полученное значение силы в точности совпадает со значением силы Лоренца.

    С учетом того, что

    можно считать, что на заряд, движущийся в магнитном поле, действует электрическое поле, направленное перпендикулярно направлению движения заряда. Этот результат также с точностью до квадратичных членов полностью совпадает с результатами концепции магнитного поля и определяется силой Лоренца.

    Как уже было сказано, одним из важных противоречий концепции магнитного поля является тот факт, что два параллельных пучка одинаковых зарядов, которые движутся с одинаковой скоростью в одном направлении, должны притягиваться.В этой модели этого противоречия уже нет. Если учесть, что скорости заряда в верхнем и нижнем проводе будут равны, а решетка отсутствует, т.е. оставить только электронные потоки, то останется только сила отталкивания.

    Таким образом, движущийся электронный поток взаимодействует одновременно как с движущимися электронами в нижнем проводе, так и с его решеткой, а сумма этих сил взаимодействия называется силой Лоренца. Сумма сил такого взаимодействия и есть сила Лоренца.

    Регулярно возникает вопрос, а создает ли магнитное поле наиболее движущийся поток электронов при отсутствии компенсирующих зарядов решетки или положительных ионов в плазме? Рассмотренная диаграмма показывает, что эффект силового взаимодействия между токоведущими системами требует в необходимом порядке наличия положительно заряженной решетки. Поэтому большая часть движущегося электронного потока не может создать тот эффект, который создается при его движении в положительно заряженной решетке.

    5. Заключение

    Показано, что зависимость скалярного потенциала заряда от скорости является физической причиной возникновения силы Лоренца. И этот результат следует считать новым законом. Эти результаты получены в предположении справедливости преобразований Галилея. Показано, что зависимость силы Лоренца от скорости нелинейна, как предполагалось ранее. Когда направления движения зарядов в проводниках совпадают, сила их взаимодействия возникает меньше, чем при разных направлениях движения.

    Список литературы

    [1] В.В. Никольский, Т.И. Никольская, Электродинамика и распространение радиоволн, Москва, Наука, 1989.
    В статье
    [2] Р. Фейнман, Р. Лейтон, М. Сендс. Лекции Фейнмана по физике, –М. Мир, т. 6 (1977).
    В статье
    [3] J.Джексон, Классическая электродинамика, Мир, Москва, 1965, 702.
    В статье
    [4] F.F. Менде, Об уточнении уравнений электромагнитной индукции, — Харьков, депонент ВИНИТИ, № 774-В88 Деп. (1988).
    В статье
    [5] Ф. Ф. Менде, Есть ли ошибки в современной физике.Харьков, Константа, 2003.
    В статье
    [6] Ф.Ф. Менде, Об уточнении некоторых законов классической электродинамики, arXiv.org/abs/physics/0402084
    В статье
    [7] FF MendeNew electrodolution. Современная физика. Харьков, НТМТ, 2012.
    В статье

    Уравнение силы Лоренца MCQ [Free PDF] — Объективный ответ на вопрос по уравнению силы Лоренца

    Сила, испытываемая электромагнитной волной в проводнике, составляет —

    .

    Опции:

    1. Электростатическая сила

    2. Магнитостатическая сила

    3. Электродвижущая сила

    4. Сила Лоренца

    Правильный ответ:

    Вариант 4 (Решение ниже)

    Решение:

    Концепция:

    Электростатическая сила: Это электрическая сила на статический заряд (Q) в проводнике из-за электрического поля ( E ), и она задается как,

    F e = E Q

    Магнитостатическая сила: Это Магнитная сила , действующая на заряд, движущийся со скоростью (В) в проводнике из-за Магнитного поля ( B ), и она задается как

    F м = Q (V × B )

    Сила Лоренца:

    Если движущийся заряд присутствует и в электрическом, и в магнитном поле, тогда сила, действующая на заряд в проводнике, известна как сила Лоренца .

    F = F e + F м

    F = E Q + Q (V × B )

    Это уравнение известно как уравнение силы Лоренца .

    Примечание : все жирные буквы — векторы.

    Итак, электромагнитная волна испытывает силу Лоренца , которая представляет собой комбинацию электростатической силы и магнитостатической силы .

    Один кулон точечного заряда, движущегося с постоянной скоростью 10 \ (\ widehat x \) м / с, входит в область x ≥ 0, имеющую плотность магнитного потока \ (\ overrightarrow B = \ left ({10y \ widehat x + 10x \ widehat y + 10 \ widehat z} \ right) T \). Величина силы, действующей на заряд при x = 0 + , составляет ______ Н.

    (\ ({\ widehat x} \), \ ({\ widehat y} \) и \ ({\ widehat z} \) — единичные векторы вдоль оси x, оси y и оси z, соответственно. 2} | = 100 \; N \)

    Уравнение силы Лоренца описывает величину силы , которую движущийся электрический заряд будет ощущать в результате присутствия магнитного поля и электрического поля E̅.

    Сила, обусловленная наличием электрического поля, определяется по формуле:

    F̅elec. = q × E̅

    q = Заряд частицы

    E̅ = вектор электрического поля

    Кроме того, сила, действующая на заряженную частицу, движущуюся в присутствии магнитного поля, определяется выражением:

    F̅mag. = q (v̅ × B̅)

    v̅ = вектор скорости частицы

    B̅ = вектор магнитного поля

    Когда заряд движется в присутствии как электрического, так и магнитного поля, сила, действующая на него, будет суммой двух сил, т.е.е.

    Fnet = F̅elec. + F̅mag.

    Fnet = q × E̅ + q (v̅ × B̅)

    Fnet = q (E̅ + v̅ × B̅)


    Опции:

    1. Движение по спирали в направлении ẑ.

    2. Круговое движение в плоскости xy.

    3. Линейное движение в направлении ẑ.

    4. Линейное движение в направлении x̂.

    Правильный ответ:

    Вариант 1 (Решение ниже)

    Решение:

    Концепция:

    Когда любой ненулевой заряд входит в магнитное поле с некоторой скоростью под углом, отличным от 90 градусов, тогда заряд следует по спиральной траектории.

    Заявка:

    \ (\ vec B = B {\ hat z_0} \)

    \ (\ vec V = {v_X} \ hat x + {v_Z} \ hat z \)

    x-компонента \ (\ vec V \) перпендикулярна магнитному полю \ (\ vec B \).

    Заряд, движущийся перпендикулярно магнитному полю, будет испытывать радиальную силу, вызывающую круговое движение, показанное на рисунке (а).

    z-компонента \ (\ vec V \) параллельна магнитному полю \ (\ vec B \). Изменение, движущееся параллельно полю, не создает силы, показанной на рисунке (b).

    Движение с компонентами, перпендикулярными и параллельными полю, заставляет изменение двигаться по спиральной траектории в направлении + z, как показано на рисунке (c).


    Вопрос 4:

    Опции:

    1. 2 r

    2. 4 r

    3. \ (\ dfrac {r} {4} \)

    4. \ (\ dfrac {r} {2} \)

      74





      Правильный ответ:

      Вариант 2 (Решение ниже)

      Решение:

      КОНЦЕПЦИЯ:

      • Сила Лоренца: основная сила, действующая на электрический заряд q, движущийся со скоростью v в магнитном поле B и электрическом поле E, называется магнитной силой Лоренца.

      Сила Лоренца — это комбинация электрической силы \ (q \ vec E \) и магнитной силы \ (q \ left ({\ vec v \ times \ vec B} \ right) \).

      • Таким образом, сила Лоренца определяется по формуле:

      \ (F = q \ left [{\ vec E + \ left ({\ vec v \ times \ vec B} \ right)} \ right] \)

      Где q — заряд, v — скорость частицы, B — магнитное поле, E — электрическое поле. {‘} = 4r \)

      • Когда скорость удваивается, а магнитное поле уменьшается вдвое, новый радиус R ‘будет в четыре раза больше, чем предыдущий радиус.Следовательно, вариант 2 верен.

      Вопрос 5:

      Сила, испытываемая электромагнитной волной в проводнике, составляет —

      .

      Опции:

      1. Электростатическая сила

      2. Магнитостатическая сила

      3. Электродвижущая сила

      4. Сила Лоренца

      Правильный ответ:

      Вариант 4 (Решение ниже)

      Решение:

      Концепция:

      Электростатическая сила: Это электрическая сила на статический заряд (Q) в проводнике из-за электрического поля ( E ), и она задается как,

      F e = E Q

      Магнитостатическая сила: Это Магнитная сила , действующая на заряд, движущийся со скоростью (В) в проводнике из-за Магнитного поля ( B ), и она задается как

      F м = Q (V × B )

      Сила Лоренца:

      Если движущийся заряд присутствует и в электрическом, и в магнитном поле, тогда сила, действующая на заряд в проводнике, известна как сила Лоренца .

      F = F e + F м

      F = E Q + Q (V × B )

      Это уравнение известно как уравнение силы Лоренца .

      Примечание : все жирные буквы — векторы.

      Итак, электромагнитная волна испытывает силу Лоренца , которая представляет собой комбинацию электростатической силы и магнитостатической силы .


      Вопрос 6:

      Опции:

      1. ноль

      2. e (v × B)

      3. e (B × v)

      4. 200 Джоуль

      Правильный ответ:

      Вариант 1 (Решение ниже)

      Решение:

      КОНЦЕПЦИЯ:

      Сила Лоренца:

      Основная сила, действующая на электрический заряд q, движущийся со скоростью v в магнитном поле B и электрическом поле E, называется магнитной силой Лоренца .

      Сила Лоренца — это комбинация электрической силы \ (q \ vec E \) и магнитной силы \ (q \ left ({\ vec v \ times \ vec B} \ right) \).

      Таким образом, сила Лоренца определяется по формуле:

      \ (⇒ F = q \ left [{\ vec E + \ left ({\ vec v \ times \ vec B} \ right)} \ right] = qE + qVB Sinθ \)

      Где q — заряд, v — скорость частицы, B — магнитное поле, E — электрическое поле.

      РАСЧЕТ:

      Задано — θ = 0 0

      Магнитная сила, действующая на объект, определяется как

      .

      ⇒ F = qVB sinθ = qVB sin 0 0 = 0 Н

      Следовательно, вариант 1 является ответом


      Вопрос 7:

      Один кулон точечного заряда, движущегося с постоянной скоростью 10 \ (\ widehat x \) м / с, входит в область x ≥ 0, имеющую плотность магнитного потока \ (\ overrightarrow B = \ left ({10y \ widehat x + 10x \ widehat y + 10 \ widehat z} \ right) T \).Величина силы, действующей на заряд при x = 0 + , составляет ______ Н.

      (\ ({\ widehat x} \), \ ({\ widehat y} \) и \ ({\ widehat z} \) — единичные векторы вдоль оси x, оси y и оси z, соответственно. )

      Ответ: 100

      Решение:

      Концепция:

      Сила , действующая на заряженную частицу, движущуюся в присутствии магнитного поля , определяется выражением:

      F̅mag = q (v̅ × B̅)

      v̅ = вектор скорости частицы

      B̅ = вектор магнитного поля

      q = Заряд частицы

      Заявка:

      Учитывая, q = 1 C

      v̅ = 10 \ (\ widehat x \) м / с

      \ (\ overrightarrow B = \ left ({10y \ widehat x + 10x \ widehat y + 10 \ widehat z} \ right) T \)

      F̅mag = q (v̅ × B̅)

      F̅mag = \ (1 (10 \; \ widehat x \ times \ left ({10y \ widehat x + 10x \ widehat y + 10 \ widehat z} \ right)) \; N \)

      mag = \ (100x \ widehat z \; + 100 (- \ widehat y) \; N \)

      F̅mag (x = 0+) = — 100 лет N

      Звездная величина F̅ mag = \ (| \ sqrt {(- 100) ^ 2} | = 100 \; N \)

      Уравнение силы Лоренца описывает величину силы , которую движущийся электрический заряд будет ощущать в результате присутствия магнитного поля и электрического поля E̅.

      Сила, обусловленная наличием электрического поля, определяется по формуле:

      F̅elec. = q × E̅

      q = Заряд частицы

      E̅ = вектор электрического поля

      Кроме того, сила, действующая на заряженную частицу, движущуюся в присутствии магнитного поля, определяется выражением:

      F̅mag. = q (v̅ × B̅)

      v̅ = вектор скорости частицы

      B̅ = вектор магнитного поля

      Когда заряд движется в присутствии как электрического, так и магнитного поля, сила, действующая на него, будет суммой двух сил, т.е.е.

      Fnet = F̅elec. + F̅mag.

      Fnet = q × E̅ + q (v̅ × B̅)

      Fnet = q (E̅ + v̅ × B̅)


      Вопрос 8:

      Точечный заряд с двумя кулонами, движущийся с равномерной скоростью 20 \ (\ widehat x \) м / с, входит в область x ≥ 0 с плотностью магнитного потока \ (\ overrightarrow B = \ left ({10y \ widehat x + 10x \ widehat y + 10 \ widehat z} \ right) T \). Величина силы, действующей на заряд при x = 0+, составляет ______ Н. 2} | = 400 \; N \)


      Уравнение силы Лоренца описывает величину силы, которую движущийся электрический заряд мог бы чувствовать в результате присутствия магнитного поля B̅ и электрического поля E̅.

      Сила, обусловленная наличием электрического поля, определяется по формуле:

      F̅elec. = q × E̅

      q = Заряд частицы

      E̅ = вектор электрического поля

      Кроме того, сила, действующая на заряженную частицу, движущуюся в присутствии магнитного поля, определяется выражением:

      F̅mag. = q (v̅ × B̅)

      v̅ = вектор скорости частицы

      B̅ = вектор магнитного поля

      Когда заряд движется в присутствии как электрического, так и магнитного поля, сила, действующая на него, будет суммой двух сил, т.е.е.

      Fnet = F̅elec. + F̅mag.

      Fnet = q × E̅ + q (v̅ × B̅)

      Fnet = q (E̅ + v̅ × B̅)


      Вопрос 9:

      Опции:

      1. ноль

      2. e (v × B)

      3. e (B × v)

      4. 200 Джоуль

      5. перпендикулярно скорости
        9057

        Правильный ответ:

        Вариант 1 (Решение ниже)

        Решение:

        КОНЦЕПЦИЯ:

        Сила Лоренца:

        Основная сила, действующая на электрический заряд q, движущийся со скоростью v в магнитном поле B и электрическом поле E, называется магнитной силой Лоренца .

        Сила Лоренца — это комбинация электрической силы \ (q \ vec E \) и магнитной силы \ (q \ left ({\ vec v \ times \ vec B} \ right) \).

        Таким образом, сила Лоренца определяется по формуле:

        \ (⇒ F = q \ left [{\ vec E + \ left ({\ vec v \ times \ vec B} \ right)} \ right] = qE + qVB Sinθ \)

        Где q — заряд, v — скорость частицы, B — магнитное поле, E — электрическое поле.

        РАСЧЕТ:

        Задано — θ = 0 0

        Магнитная сила, действующая на объект, определяется как

        .

        ⇒ F = qVB sinθ = qVB sin 0 0 = 0 Н

        Следовательно, вариант 1 является ответом


        Вопрос 10:

        Сила, испытываемая электромагнитной волной в проводнике, составляет —

        .

        Опции:

        1. Электростатическая сила

        2. Магнитостатическая сила

        3. Электродвижущая сила

        4. Сила Лоренца



        5. сила тяжести

          Правильный ответ:

          Вариант 4 (Решение ниже)

          Решение:

          Концепция:

          Электростатическая сила: Это электрическая сила на статический заряд (Q) в проводнике из-за электрического поля ( E ), и она задается как,

          F e = E Q

          Магнитостатическая сила: Это Магнитная сила , действующая на заряд, движущийся со скоростью (В) в проводнике из-за Магнитного поля ( B ), и она задается как

          F м = Q (V × B )

          Сила Лоренца:

          Если движущийся заряд присутствует и в электрическом, и в магнитном поле, тогда сила, действующая на заряд в проводнике, известна как сила Лоренца .

          F = F e + F м

          F = E Q + Q (V × B )

          Это уравнение известно как уравнение силы Лоренца .

          Примечание : все жирные буквы — векторы.

          Итак, электромагнитная волна испытывает силу Лоренца , которая представляет собой комбинацию электростатической силы и магнитостатической силы .


          .

    alexxlab

    Добавить комментарий

    Ваш адрес email не будет опубликован. Обязательные поля помечены *